diff --git "a/data/en/en.dev_casimedicos.jsonl" "b/data/en/en.dev_casimedicos.jsonl" new file mode 100644--- /dev/null +++ "b/data/en/en.dev_casimedicos.jsonl" @@ -0,0 +1,63 @@ +{"id": 274, "year": 2016, "question_id_specific": 73, "full_question": "A 52-year-old man with no concomitant diseases comes to the emergency department for melena of 24 hours evolution without hemodynamic repercussions. He denies taking nonsteroidal anti-inflammatory drugs. The hematocrit is 33% and the rest of the laboratory tests are normal. The upper gastrointestinal endoscopy performed urgently 6 hours after admission shows a normal stomach, without blood or hematic debris and an excavated ulcer of 8 mm in diameter in the anterior face of the duodenal bulb with \"visible vessel\" at its base and without active bleeding. Which of the following statements is true?", "full_answer": "This is a grade IIa ulcer (Forrest classification), with a high risk of recurrence. Therefore, endoscopic treatment and hospitalization with intravenous treatment with PPIs (in our setting it is usually omeprazole) for at least 72 hours is clearly indicated. Surgery is indicated in massive bleeding or bleeding refractory to medical treatment.", "type": "DIGESTIVE SYSTEM", "options": {"1": "In the initial endoscopy it is indicated to apply an endoscopic therapy and subsequently to establish endovenous treatment with high doses of a proton pump inhibitor. This strategy has been shown to reduce the risk of hemorrhagic recurrence and mortality.", "2": "In the initial endoscopy, given the absence of active bleeding, endoscopic therapy is not indicated. Subsequently, to reduce the risk of hemorrhagic recurrence, intravenous treatment with high doses of a proton pump inhibitor should be started.", "3": "n initial endoscopy, endoscopic therapy is indicated. Subsequent treatment with high doses of a proton pump inhibitor has not been shown to be of any additional benefit.", "4": "Since this is a complicated ulcer (hemorrhage) the best therapeutic option, once the hemorrhagic episode is resolved, is a vagotomy and pyloroplasty.", "5": NaN}, "correct_option": 1, "explanations": {"1": {"exist": true, "char_ranges": [[0, 258]], "word_ranges": [[0, 39]], "text": "This is a grade IIa ulcer (Forrest classification), with a high risk of recurrence. Therefore, endoscopic treatment and hospitalization with intravenous treatment with PPIs (in our setting it is usually omeprazole) for at least 72 hours is clearly indicated."}, "2": {"exist": false, "char_ranges": [], "word_ranges": [], "text": ""}, "3": {"exist": false, "char_ranges": [], "word_ranges": [], "text": ""}, "4": {"exist": true, "char_ranges": [[259, 344]], "word_ranges": [[39, 51]], "text": "Surgery is indicated in massive bleeding or bleeding refractory to medical treatment."}, "5": {"exist": false, "char_ranges": [], "word_ranges": [], "text": ""}}} +{"id": 233, "year": 2014, "question_id_specific": 104, "full_question": "In a 29-year-old woman the following parameters are found in the course of a routine blood test: Hb 11.5 g/dL, MCV 70 fl, MCH 28 pg, Ferritin 10 ng/mL, leukocytes 5,200/mm3, platelets 335,000/mm3. Physical examination is normal. The most indicated examination in this situation is:", "full_answer": "The most frequent cause of microcytic iron deficiency anemia in young women is chronic gynecologic leaks.", "type": "HEMATOLOGY", "options": {"1": "Gynecological examination.", "2": "Fecal occult blood test.", "3": "Radiological study of the digestive tract.", "4": "Hemoglobin electrophoresis.", "5": "Cooms test."}, "correct_option": 1, "explanations": {"1": {"exist": true, "char_ranges": [[0, 105]], "word_ranges": [[0, 16]], "text": "The most frequent cause of microcytic iron deficiency anemia in young women is chronic gynecologic leaks."}, "2": {"exist": false, "char_ranges": [], "word_ranges": [], "text": ""}, "3": {"exist": false, "char_ranges": [], "word_ranges": [], "text": ""}, "4": {"exist": false, "char_ranges": [], "word_ranges": [], "text": ""}, "5": {"exist": false, "char_ranges": [], "word_ranges": [], "text": ""}}} +{"id": 315, "year": 2016, "question_id_specific": 136, "full_question": "A 41-year-old man comes to the Emergency Department for three days of swelling and pain in the right knee, with functional impotence and fever Two weeks earlier he had had a self-inflicted diarrhea. On examination there is joint effusion, so we proceed to perform an arthrocentesis and obtain 50 cc of cloudy fluid, with decreased viscosity and the following analytical parameters: leukocytes 40. 000/microL (85% of neutrophils), glucose 40 mg/dL, absence of crystals, Gram stain: no microorganisms are observed. Which of the following statements about this patient is WRONG:", "full_answer": "A negative gram or culture never rules out the option of infectious etiology, even less in possible septic arthritis. All others are true.", "type": "RHEUMATOLOGY", "options": {"1": "Treatment with cloxacillin and ceftriaxone should be initiated pending the results of the fluid culture.", "2": "It is advisable to perform daily aitrocentesis to alleviate symptoms and avoid joint destruction.", "3": "If the culture is negative, it is likely to be reactive arthritis.", "4": "Negative Gram stain rules out septic arthritis.", "5": NaN}, "correct_option": 4, "explanations": {"1": {"exist": false, "char_ranges": [], "word_ranges": [], "text": ""}, "2": {"exist": false, "char_ranges": [], "word_ranges": [], "text": ""}, "3": {"exist": false, "char_ranges": [], "word_ranges": [], "text": ""}, "4": {"exist": true, "char_ranges": [[0, 117]], "word_ranges": [[0, 19]], "text": "A negative gram or culture never rules out the option of infectious etiology, even less in possible septic arthritis."}, "5": {"exist": false, "char_ranges": [], "word_ranges": [], "text": ""}}} +{"id": 235, "year": 2014, "question_id_specific": 107, "full_question": "A 71-year-old man presenting with severe pancytopenia without presence of immature cells and with a bone marrow study suggestive of severe aplastic anemia. What would be the fundamental therapeutic approach?", "full_answer": "Severe aplastic anemia and not a candidate for allogeneic transplant because of his age. Antisuppressants seem to be the most suitable.", "type": "HEMATOLOGY", "options": {"1": "Treatment with methylprednisolone at a dose of 1 g/kg/day for 5 days.", "2": "Study of siblings and if any of them is HLA compatible, allogeneic transplantation of hematopoietic progenitors.", "3": "Immunosuppressive therapy with cyclosporine and antithymocyte immunoglobulin.", "4": "Hemotherapy support.", "5": "Chemotherapy and if response autologous transplantation of hematopoietic progenitors."}, "correct_option": 3, "explanations": {"1": {"exist": false, "char_ranges": [], "word_ranges": [], "text": ""}, "2": {"exist": false, "char_ranges": [], "word_ranges": [], "text": ""}, "3": {"exist": true, "char_ranges": [[0, 135]], "word_ranges": [[0, 21]], "text": "Severe aplastic anemia and not a candidate for allogeneic transplant because of his age. Antisuppressants seem to be the most suitable."}, "4": {"exist": false, "char_ranges": [], "word_ranges": [], "text": ""}, "5": {"exist": false, "char_ranges": [], "word_ranges": [], "text": ""}}} +{"id": 429, "year": 2018, "question_id_specific": 114, "full_question": "A 34-year-old man consults the emergency department for dysuria and burning in the urethral meatus, with the presence of a thick, whitish discharge coming out through the meatus for the past three days. Which of the following answers is false?", "full_answer": "A microbiologic confirmatory test of cure is not needed for persons with a diagnosis of uncomplicated urogenital or rectal gonorrhea who are treated with any of the recommended or alternative regimens; however, anyone with pharyngeal gonorrhea who is treated with an alternative regimen should return 14 days after treatment for a test of cure using culture or a nucleic acid amplification test (NAAT). If the HAT is positive, a confirmatory culture should be attempted prior to retreatment. Antimicrobial susceptibility testing should be performed on all positive cultures. Symptoms that persist after treatment should be evaluated by culture for N. gonorrhoeae (with or without concurrent TAAN), and any gonococcal isolates should be tested to identify antimicrobial susceptibility (modified from Sexually Transmitted Diseases Treatment Guidelines, 2015).", "type": "INFECTIOUS DISEASES AND MICROBIOLOGY", "options": {"1": "A sample of urethral exudate should be obtained to make the etiological diagnosis.", "2": "Empirical treatment with intramuscular ceftriaxone and oral azithromycin in single doses, pending microbiological results.", "3": "Antibiogram should be performed in cases where gonorrhoeae is isolated.", "4": "2-3 weeks after treatment, new urethral samples should be obtained to confirm microbiological cure.", "5": NaN}, "correct_option": 4, "explanations": {"1": {"exist": false, "char_ranges": [], "word_ranges": [], "text": ""}, "2": {"exist": false, "char_ranges": [], "word_ranges": [], "text": ""}, "3": {"exist": false, "char_ranges": [], "word_ranges": [], "text": ""}, "4": {"exist": true, "char_ranges": [[211, 402]], "word_ranges": [[32, 63]], "text": "anyone with pharyngeal gonorrhea who is treated with an alternative regimen should return 14 days after treatment for a test of cure using culture or a nucleic acid amplification test (NAAT)."}, "5": {"exist": false, "char_ranges": [], "word_ranges": [], "text": ""}}} +{"id": 540, "year": 2021, "question_id_specific": 4, "full_question": "A 40-year-old patient begins to feel discomfort in the right buttock after a prolonged car trip. Since then, he does not tolerate prolonged sitting due to recurrence of pain. Occasionally he feels numbness in the right lower limb that disappears when he gets up. He has a magnetic resonance imaging study and a hip X-ray, in which no abnormalities are reported. Examination of sensibility and osteotendinous reflexes shows no alterations. External rotation maneuvers and resisted abduction, with the hip in 90 degrees of flexion, reproduce the pain. The most probable diagnosis is:", "full_answer": "Piriformis syndrome occurs due to compression or impingement of the sciatic nerve by hypertrophy or contracture of the piriformis muscle. The characteristic symptoms of pyramidal syndrome include gluteal pain radiating to the posterior aspect of the thigh that may be accompanied by paresthesias. As a frequent trigger, the patient may report a long period of sitting on a hard surface prior to the pain. The clinical case presents the PACE maneuver in which the patient resists the examiner's external rotation and hip abduction to 90° of flexion. It is considered positive if it reproduces the symptoms in the context of pyramidal syndrome.", "type": "TRAUMATOLOGY", "options": {"1": "Femoroacetabular impingement syndrome.", "2": "Tensor fascia lata syndrome.", "3": "S1 radiculopathy.", "4": "Pyramidal syndrome.", "5": NaN}, "correct_option": 4, "explanations": {"1": {"exist": false, "char_ranges": [], "word_ranges": [], "text": ""}, "2": {"exist": false, "char_ranges": [], "word_ranges": [], "text": ""}, "3": {"exist": false, "char_ranges": [], "word_ranges": [], "text": ""}, "4": {"exist": true, "char_ranges": [[405, 642]], "word_ranges": [[64, 102]], "text": "The clinical case presents the PACE maneuver in which the patient resists the examiner's external rotation and hip abduction to 90° of flexion. It is considered positive if it reproduces the symptoms in the context of pyramidal syndrome."}, "5": {"exist": false, "char_ranges": [], "word_ranges": [], "text": ""}}} +{"id": 266, "year": 2014, "question_id_specific": 138, "full_question": "In a patient with mild chronic obstructive pulmonary disease (COPD) in stable phase, polyglobulia and respiratory insufficiency are evident. In the chest X-ray there are no notable alterations. Which of the following procedures would you consider performing first to rule out the coexistence of other diseases that may justify the findings described?", "full_answer": "The approach is questionable due to lack of sufficient data. The correct answer could be respiratory polygraphy, especially in the case of an obese patient, which we do not know, but it could also be correct 2 (echocardiogram) in the case of suspicion of associated cardiopathy, less frequent but also possible. The association of COPD with OSAHS is a frequent cause of COPD deterioration, greater than that which would correspond to functional impairment.", "type": "PNEUMOLOGY", "options": {"1": "Respiratory polygraphy.", "2": "Echocardiogram.", "3": "Thoracic computed tomography.", "4": "Pulmonary scintigraphy.", "5": "Bone marrow puncture."}, "correct_option": 1, "explanations": {"1": {"exist": true, "char_ranges": [[61, 112]], "word_ranges": [[10, 17]], "text": "The correct answer could be respiratory polygraphy,"}, "2": {"exist": true, "char_ranges": [[183, 311]], "word_ranges": [[31, 51]], "text": "it could also be correct 2 (echocardiogram) in the case of suspicion of associated cardiopathy, less frequent but also possible."}, "3": {"exist": false, "char_ranges": [], "word_ranges": [], "text": ""}, "4": {"exist": false, "char_ranges": [], "word_ranges": [], "text": ""}, "5": {"exist": false, "char_ranges": [], "word_ranges": [], "text": ""}}} +{"id": 13, "year": 2011, "question_id_specific": 150, "full_question": "A one-and-a-half-year-old boy presents to the emergency department with abdominal pain and jaundice. On examination, an abdominal mass is palpated. An ultrasound shows the presence of a common bile duct cyst. What therapeutic approach should we adopt?", "full_answer": "I believe that the correct answer is 4 the common bile duct cyst is a rare pathology but its treatment is surgical and consists of resection of the cyst and hepaticojejunosotomy. Cholangioresonance is necessary to choose the appropriate technique for the four existing classes of common bile duct cyst (Todani classification).", "type": "SURGERY", "options": {"1": "A percutaneous puncture will be performed for peritoneal lavage to check that the cyst contains bile in order to leave a drainage to improve abdominal pain.", "2": "Radical resection of the entire bile duct is indicated to replace it with a loop of bowel.", "3": "An exploratory laparotomy and drainage of the cyst will be indicated and when the dilatation subsides the drainage will be removed.", "4": "A cholangioresonance will be performed to delimit the cyst and a laparotomy will be indicated for resection of the cyst and anastomosis of the biliary tract.", "5": "HIDA scintigraphy is necessary to delimit the cyst and to be able to perform percutaneous drainage safely."}, "correct_option": 4, "explanations": {"1": {"exist": false, "char_ranges": [], "word_ranges": [], "text": ""}, "2": {"exist": false, "char_ranges": [], "word_ranges": [], "text": ""}, "3": {"exist": false, "char_ranges": [], "word_ranges": [], "text": ""}, "4": {"exist": true, "char_ranges": [[0, 326]], "word_ranges": [[0, 51]], "text": "I believe that the correct answer is 4 the common bile duct cyst is a rare pathology but its treatment is surgical and consists of resection of the cyst and hepaticojejunosotomy. Cholangioresonance is necessary to choose the appropriate technique for the four existing classes of common bile duct cyst (Todani classification)."}, "5": {"exist": false, "char_ranges": [], "word_ranges": [], "text": ""}}} +{"id": 299, "year": 2016, "question_id_specific": 104, "full_question": "A 46-year-old male parenteral drug user came to the emergency department with fever with chills, mental confusion, diffuse myalgias and intense pain in the left hand of 24 hours of evolution without clear local phlogosis signs. She does not refer any traumatic history. Examination revealed: temperature 38.9°C, 120 beats per minute, respiratory rate 30/min, blood pressure 90/54 mm Hg. Laboratory tests showed leukocytosis with left deviation (25,000 leukocytes/mm3, 80% neutrophils); increased creatinine (1.6 mg/dL) and CK (138 U/L). Among the following, which is the most likely diagnosis:", "full_answer": "I have my doubts between 1 and 2; exceptionally C. botulinum can infect traumatic or puncture wounds in IVDUs using brown heroin or cause sinusitis in cocaine addicts. On the other hand, the case may refer to a severe sepsis with poor initial manifestation at first sight, with incipient rhabdomyolysis and muscle pain, in this case it may be a necrotizing fasciitis, where the most frequently implicated germ is S. pyogenes.", "type": "INFECTIOUS DISEASES", "options": {"1": "Streptococcal necrotizing fasciitis.", "2": "Gangrene due to Clostridium spp.", "3": "Mycobacterial cellulitis.", "4": "Erysipelas.", "5": NaN}, "correct_option": 1, "explanations": {"1": {"exist": true, "char_ranges": [[187, 367]], "word_ranges": [[32, 62]], "text": "the case may refer to a severe sepsis with poor initial manifestation at first sight, with incipient rhabdomyolysis and muscle pain, in this case it may be a necrotizing fasciitis,"}, "2": {"exist": true, "char_ranges": [[34, 167]], "word_ranges": [[8, 28]], "text": "exceptionally C. botulinum can infect traumatic or puncture wounds in IVDUs using brown heroin or cause sinusitis in cocaine addicts."}, "3": {"exist": false, "char_ranges": [], "word_ranges": [], "text": ""}, "4": {"exist": false, "char_ranges": [], "word_ranges": [], "text": ""}, "5": {"exist": false, "char_ranges": [], "word_ranges": [], "text": ""}}} +{"id": 16, "year": 2011, "question_id_specific": 131, "full_question": "A 62-year-old man with a history of significant alcohol abuse, carrier of hepatitis C virus, treated with Ibuprofen for tendinitis of the right shoulder, goes to his dermatologist because after spending two weeks on vacation at the beach he notices the appearance of tense blisters on the dorsum of his hands. On examination, in addition to localization and slight malar hypertrichosis. The most likely diagnosis is:", "full_answer": "Porphyria Cutanea Tarda: 60% of patients with PCT are male, many of them drink alcohol in excess, women who develop it are usually treated with drugs containing estrogens. Most are males with signs of iron overload, this overload reduces the activity of the enzyme uroporphyrinogen decarboxylase, which leads to the elevation of uroporphyrins. HCV and HIV infections have been implicated in the precipitation of acquired PCT. There is a hereditary form with AD pattern. Patients with PCT present with blistering of photoexposed skin, most frequently on the dorsum of the hands and scalp. In addition to fragility, they may develop hypertrichosis, hyperpigmentation, cicatricial alopecia and sclerodermal induration.", "type": "DERMATOLOGY", "options": {"1": "Epidermolysis bullosa acquisita.", "2": "Porphyria cutanea tarda.", "3": "Phototoxic reaction.", "4": "Contact dermatitis.", "5": "Acute intermittent porphyria."}, "correct_option": 2, "explanations": {"1": {"exist": false, "char_ranges": [], "word_ranges": [], "text": ""}, "2": {"exist": true, "char_ranges": [[0, 342]], "word_ranges": [[0, 53]], "text": "Porphyria Cutanea Tarda: 60% of patients with PCT are male, many of them drink alcohol in excess, women who develop it are usually treated with drugs containing estrogens. Most are males with signs of iron overload, this overload reduces the activity of the enzyme uroporphyrinogen decarboxylase, which leads to the elevation of uroporphyrins."}, "3": {"exist": false, "char_ranges": [], "word_ranges": [], "text": ""}, "4": {"exist": false, "char_ranges": [], "word_ranges": [], "text": ""}, "5": {"exist": false, "char_ranges": [], "word_ranges": [], "text": ""}}} +{"id": 121, "year": 2012, "question_id_specific": 114, "full_question": "47-year-old woman. Native of Cochabamba (Bolivia), she has been living in Spain for 8 years. In mid 2009 she began to experience palpitations, dyspnea and progressive edema in the lower extremities. She was diagnosed with heart failure secondary to cardiomyopathy due to Chagas disease. What is the etiological agent of this disease endemic in several Latin American countries?", "full_answer": "The correct answer is the fifth one. The cause of Chagas disease is Trypanosoma cruzi.", "type": "MICROBIOLOGY", "options": {"1": "Toxoplasma gondii.", "2": "Trypanosoma brucei.", "3": "Lehismania donovani.", "4": "Giardia lamblia.", "5": "Trypanosoma cruzi."}, "correct_option": 5, "explanations": {"1": {"exist": false, "char_ranges": [], "word_ranges": [], "text": ""}, "2": {"exist": false, "char_ranges": [], "word_ranges": [], "text": ""}, "3": {"exist": false, "char_ranges": [], "word_ranges": [], "text": ""}, "4": {"exist": false, "char_ranges": [], "word_ranges": [], "text": ""}, "5": {"exist": true, "char_ranges": [[37, 86]], "word_ranges": [[7, 15]], "text": "The cause of Chagas disease is Trypanosoma cruzi."}}} +{"id": 241, "year": 2014, "question_id_specific": 229, "full_question": "A 29-year-old obese woman, with no past history of interest, consults for a month of intense, bilateral, pulsating headache, accompanied by horizontal diplopia and episodes of monocular amaurosis lasting seconds. Examination is normal, except for the presence of bilateral papilledema. Which of these tests do you think will allow you to fully confirm the diagnosis?", "full_answer": "Again we are presented with a case to make a diagnosis and then, in this case, perform a confirmatory test. Now the guiding symptom is bilateral papilledema which is indicative of intracranial hypertension. Although he does not comment that a space-occupying lesion has been ruled out, given the normal neurological examination and the other symptoms, it is deduced that it is idiopathic intracranial hypertension or pseudotumor cerebri. Other data suggestive of this entity include: women of reproductive age, obesity, and diplopia (due to involvement of the VI cranial nerve). It is characteristic that the headache and visual blurriness suddenly worsen in the mornings when getting up or changing posture, producing transitory blindness (Neurology, Zarranz). The lumbar puncture invariably shows an increase in cerebrospinal fluid pressure, so the correct answer is 4.", "type": "NEUROLOGY", "options": {"1": "Ultrasound of supra-aortic trunks.", "2": "Cranial magnetic resonance imaging.", "3": "Electroencephalogram.", "4": "Lumbar puncture.", "5": "Visual evoked potentials."}, "correct_option": 4, "explanations": {"1": {"exist": false, "char_ranges": [], "word_ranges": [], "text": ""}, "2": {"exist": false, "char_ranges": [], "word_ranges": [], "text": ""}, "3": {"exist": false, "char_ranges": [], "word_ranges": [], "text": ""}, "4": {"exist": true, "char_ranges": [[579, 871]], "word_ranges": [[89, 131]], "text": "It is characteristic that the headache and visual blurriness suddenly worsen in the mornings when getting up or changing posture, producing transitory blindness (Neurology, Zarranz). The lumbar puncture invariably shows an increase in cerebrospinal fluid pressure, so the correct answer is 4."}, "5": {"exist": false, "char_ranges": [], "word_ranges": [], "text": ""}}} +{"id": 576, "year": 2022, "question_id_specific": 103, "full_question": "A 30-year-old male who consults for unilateral periocular headaches that rapidly progress to become very intense. The pain attacks last 30-60 minutes, occur 2-3 times a day, and are accompanied by tearing in the painful eye and unilateral rhinorrhea. What is the most appropriate preventive treatment?", "full_answer": "They describe a characteristic case of cluster headache. It is more frequent in males, the pain is characteristically unilateral periocular of great intensity, of variable duration, usually minutes, and it associates trigeminoautonomic signs such as rhinorrhea and unilateral conjunctival injection. The preventive treatment par excellence is verapamil associated with a descending corticosteroid regimen, as described in answer 2.", "type": "NEUROLOGY", "options": {"1": "Oral prednisone with withdrawal in 2-3 months.", "2": "Oral prednisone with withdrawal in one month, associated with oral verapamil at a dose of 80-180 mg every 8 hours.", "3": "Carbamazepine at doses of 600-1200 mg per day.", "4": "Indomethacin at a dose of 50 mg every 8 hours for one month.", "5": NaN}, "correct_option": 2, "explanations": {"1": {"exist": false, "char_ranges": [], "word_ranges": [], "text": ""}, "2": {"exist": true, "char_ranges": [[0, 431]], "word_ranges": [[0, 58]], "text": "They describe a characteristic case of cluster headache. It is more frequent in males, the pain is characteristically unilateral periocular of great intensity, of variable duration, usually minutes, and it associates trigeminoautonomic signs such as rhinorrhea and unilateral conjunctival injection. The preventive treatment par excellence is verapamil associated with a descending corticosteroid regimen, as described in answer 2."}, "3": {"exist": false, "char_ranges": [], "word_ranges": [], "text": ""}, "4": {"exist": false, "char_ranges": [], "word_ranges": [], "text": ""}, "5": {"exist": false, "char_ranges": [], "word_ranges": [], "text": ""}}} +{"id": 504, "year": 2020, "question_id_specific": 87, "full_question": "Given sudden-onset obsessive-compulsive disorder (OCD) in a 9-year-old boy who also has tics and chorea as neurologic manifestations, an infection by should be ruled out:", "full_answer": "We are presented with a case of PANDAS syndrome, a rare complication of an infection caused by group A beta-hemolytic streptococcus.", "type": "PEDIATRICS", "options": {"1": "Streptococcus viridans.", "2": "Alpha-hemolytic streptococcus.", "3": "Beta-hemolytic streptococcus, group A.", "4": "Enterococcus.", "5": NaN}, "correct_option": 3, "explanations": {"1": {"exist": false, "char_ranges": [], "word_ranges": [], "text": ""}, "2": {"exist": false, "char_ranges": [], "word_ranges": [], "text": ""}, "3": {"exist": true, "char_ranges": [[0, 132]], "word_ranges": [[0, 21]], "text": "We are presented with a case of PANDAS syndrome, a rare complication of an infection caused by group A beta-hemolytic streptococcus."}, "4": {"exist": false, "char_ranges": [], "word_ranges": [], "text": ""}, "5": {"exist": false, "char_ranges": [], "word_ranges": [], "text": ""}}} +{"id": 367, "year": 2016, "question_id_specific": 119, "full_question": "A 38-year-old nonsmoking woman previously diagnosed with atopic rhinitis presents with a two-month history of intermittent cough and wheezing, predominantly at night. Chest X-ray was normal. Spirometry within the normal range with negative bronchodilator test. Which of the following tests would you request next?", "full_answer": "A normal spirometry and a negative bronchodilator test does not exclude asthma, and the patient's clinical picture is highly suggestive of asthma, so a methacholine test would confirm the diagnosis, at least of bronchial hyperresponsiveness.", "type": "PNEUMOLOGY AND THORACIC SURGERY", "options": {"1": "Paranasal sinus Rx.", "2": "Thoracic CT.", "3": "Methacholine test.", "4": "Prick test.", "5": NaN}, "correct_option": 3, "explanations": {"1": {"exist": false, "char_ranges": [], "word_ranges": [], "text": ""}, "2": {"exist": false, "char_ranges": [], "word_ranges": [], "text": ""}, "3": {"exist": true, "char_ranges": [[0, 241]], "word_ranges": [[0, 35]], "text": "A normal spirometry and a negative bronchodilator test does not exclude asthma, and the patient's clinical picture is highly suggestive of asthma, so a methacholine test would confirm the diagnosis, at least of bronchial hyperresponsiveness."}, "4": {"exist": false, "char_ranges": [], "word_ranges": [], "text": ""}, "5": {"exist": false, "char_ranges": [], "word_ranges": [], "text": ""}}} +{"id": 444, "year": 2018, "question_id_specific": 104, "full_question": "A 36-year-old woman comes to the clinic because she has had three miscarriages in the first trimester. She has not had any full-term pregnancies. The studies that you will request in the first term are NOT included:", "full_answer": "The correct answer is 1. This is because hysterosalpingography determines if there is any obstruction of the tubes and, therefore, fertilization does not occur. This is not the case in our patient, where fertilization does occur and even implantation occurs. The rest of the diagnostic tests would be included.", "type": "GYNECOLOGY AND OBSTETRICS", "options": {"1": "Hysterosalpingography.", "2": "Peripheral blood karyotyping of both partners.", "3": "Determination of antiphospholipid antibodies.", "4": "Vaginal ultrasound.", "5": NaN}, "correct_option": 1, "explanations": {"1": {"exist": true, "char_ranges": [[0, 258]], "word_ranges": [[0, 40]], "text": "The correct answer is 1. This is because hysterosalpingography determines if there is any obstruction of the tubes and, therefore, fertilization does not occur. This is not the case in our patient, where fertilization does occur and even implantation occurs."}, "2": {"exist": true, "char_ranges": [[259, 310]], "word_ranges": [[40, 49]], "text": "The rest of the diagnostic tests would be included."}, "3": {"exist": true, "char_ranges": [[259, 310]], "word_ranges": [[40, 49]], "text": "The rest of the diagnostic tests would be included."}, "4": {"exist": true, "char_ranges": [[259, 310]], "word_ranges": [[40, 49]], "text": "The rest of the diagnostic tests would be included."}, "5": {"exist": false, "char_ranges": [], "word_ranges": [], "text": ""}}} +{"id": 171, "year": 2013, "question_id_specific": 226, "full_question": "A 26-year-old male patient, competitive athlete, is studied because his brother has had a sudden death. Complementary tests detect the existence of obstructive hypertrophic cardiomyopathy. Mark the FALSE option regarding this pathology:", "full_answer": "Our myocardium is thicker than it should be, predominantly in the septum, and it is so thick that it obstructs the LV outflow, producing a \"functional\" stenosis with a pressure gradient. Our objective will be, first of all, to \"relax\" the myocardium: for this purpose, a beta-blocker is recommended, whereas digoxin, as a good positive inotrope, increases the gradient and is contraindicated. Sport is not highly recommended, given the increased risk of sudden death associated with this pathology. And if we have a fat septum, which protrudes into the LV cavity, it is not advisable to perform volume depletion, since we would also be increasing the gradient. Regarding number 5, it is true: it is convenient to perform a careful exploration and an ECG in the relatives of patients with hypertrophy.", "type": "CARDIOLOGY AND CARDIOVASCULAR SURGERY", "options": {"1": "Treatment with beta-blockers helps to reduce symptoms.", "2": "Abandonment of competitive sport should be recommended.", "3": "Digoxin is useful to reduce the subaortic gradient.", "4": "Diuretics should be avoided or used with great care.", "5": "Given the hereditary nature of the pathology, a screening study should be performed in the rest of the direct relatives."}, "correct_option": 3, "explanations": {"1": {"exist": true, "char_ranges": [[186, 299]], "word_ranges": [[30, 49]], "text": "gradient. Our objective will be, first of all, to \"relax\" the myocardium: for this purpose, a beta-blocker is recommended,"}, "2": {"exist": true, "char_ranges": [[393, 498]], "word_ranges": [[62, 78]], "text": "Sport is not highly recommended, given the increased risk of sudden death associated with this pathology."}, "3": {"exist": true, "char_ranges": [[300, 392]], "word_ranges": [[49, 62]], "text": "whereas digoxin, as a good positive inotrope, increases the gradient and is contraindicated."}, "4": {"exist": true, "char_ranges": [[503, 660]], "word_ranges": [[79, 107]], "text": "if we have a fat septum, which protrudes into the LV cavity, it is not advisable to perform volume depletion, since we would also be increasing the gradient."}, "5": {"exist": true, "char_ranges": [[661, 800]], "word_ranges": [[107, 131]], "text": "Regarding number 5, it is true: it is convenient to perform a careful exploration and an ECG in the relatives of patients with hypertrophy."}}} +{"id": 555, "year": 2022, "question_id_specific": 51, "full_question": "55-year-old woman with type 1 diabetes diagnosed at age 15 years who refers nausea and abdominal distention, especially after meals. Evaluation is consistent with diabetic gastroparesis. Which of the following medications would be the most appropriate treatment to improve her symptoms:", "full_answer": "It was approved as a treatment for gastroparesis by the US Food and Drug Administration (FDA) in 1979 and remains the first-line drug. It acts as a prokinetic by its antagonistic effect on dopamine 2 (D2) receptors promoting gastric emptying and, in addition, binds to the 5-hydroxytryptamine 4 receptor (serotonin 5-HT4) to stimulate cholinergic nerve pathways in the stomach. Physiologically, it accelerates intestinal transit by increasing the tone and amplitude of gastric contractions, increases lower esophageal sphincter pressure and improves antropyloroduodenal coordination. In addition, this antiemetic agent provides relief through antagonism of central and peripheral dopamine receptors.", "type": "ENDOCRINOLOGY", "options": {"1": "Lansoprazole.", "2": "Ondansetron.", "3": "Loperamide.", "4": "Metoclopramide.", "5": NaN}, "correct_option": 4, "explanations": {"1": {"exist": false, "char_ranges": [], "word_ranges": [], "text": ""}, "2": {"exist": false, "char_ranges": [], "word_ranges": [], "text": ""}, "3": {"exist": false, "char_ranges": [], "word_ranges": [], "text": ""}, "4": {"exist": true, "char_ranges": [[0, 134]], "word_ranges": [[0, 23]], "text": "It was approved as a treatment for gastroparesis by the US Food and Drug Administration (FDA) in 1979 and remains the first-line drug."}, "5": {"exist": false, "char_ranges": [], "word_ranges": [], "text": ""}}} +{"id": 320, "year": 2016, "question_id_specific": 142, "full_question": "A 40-year-old man, with no history of interest, presents with acute arthritis of the right knee of 5 days of evolution as well as fever up to 37.7°C. He has no history of trauma. He presented with joint effusion, warmth on palpation and functional limitation due to pain. Diagnostic arthrocentesis was performed and showed 30000 cels/microliter with predominance of neutrophils. Polarized light examination revealed rhomboidal and cylindrical crystals with weakly positive birefringence. On plain radiograph of the knee, radiodense deposits are found in the external meniscus of the knee. Mark the FALSE answer:", "full_answer": "It is a chondrocalcinosis. There is no treatment for this disease, only treatment for flare-ups (colchicine, NSAIDs or corticosteroids), although sometimes if the flare-ups are habitual the colchicine is maintained. Allopurinol has not shown any efficacy in this microcrystal arthropathy, but it has shown efficacy in gout.", "type": "RHEUMATOLOGY", "options": {"1": "Initial treatment will consist of non-steroidal anti-inflammatory drugs, as well as aspiration of synovial fluid.", "2": "It is advisable to perform a synovial fluid culture to rule out infectious synovitis.", "3": "Once the acute flare-up has passed, treatment with allopurinol should be started to reduce the incidence of new arthritis flare-ups in the future.", "4": "The most likely diagnosis is acute arthritis due to calcium pyrophosphate crystal deposition (pseudogout).", "5": NaN}, "correct_option": 3, "explanations": {"1": {"exist": true, "char_ranges": [[27, 136]], "word_ranges": [[4, 19]], "text": "There is no treatment for this disease, only treatment for flare-ups (colchicine, NSAIDs or corticosteroids),"}, "2": {"exist": false, "char_ranges": [], "word_ranges": [], "text": ""}, "3": {"exist": true, "char_ranges": [[216, 323]], "word_ranges": [[30, 47]], "text": "Allopurinol has not shown any efficacy in this microcrystal arthropathy, but it has shown efficacy in gout."}, "4": {"exist": true, "char_ranges": [[216, 323]], "word_ranges": [[30, 47]], "text": "Allopurinol has not shown any efficacy in this microcrystal arthropathy, but it has shown efficacy in gout."}, "5": {"exist": false, "char_ranges": [], "word_ranges": [], "text": ""}}} +{"id": 617, "year": 2022, "question_id_specific": 119, "full_question": "13-year-old female, with no relevant history, with menarche 3 months ago, followed since the age of 10 years for idiopathic scoliosis that has worsened. In the physical examination she presents a hump of 7 degrees in the Adams test and in the scoliogram a T4-L1 thoracolumbar curve of 35 degrees of Cobb and a Risser 0:", "full_answer": "We are presented with the case of a 13 year old patient, with menarche 3 months ago, a T4-L1 thoracolumbar curve of 35º and a Risser 0. As a general rule we could say that below 50º the treatment is usually conservative, with two options: observation or use of a brace to slow down the evolution. In this case, taking into account the remaining growth (menarche 3 months ago and Risser 0) the most indicated would be to use a brace (Answer 2 correct).", "type": "TRAUMATOLOGY", "options": {"1": "Recommend swimming and revision in three months.", "2": "Prescribe a brace type orthosis.", "3": "Refer to physiotherapy for spine elastification.", "4": "Revise in 6 months with a new X-ray.", "5": NaN}, "correct_option": 2, "explanations": {"1": {"exist": false, "char_ranges": [], "word_ranges": [], "text": ""}, "2": {"exist": true, "char_ranges": [[0, 451]], "word_ranges": [[0, 84]], "text": "We are presented with the case of a 13 year old patient, with menarche 3 months ago, a T4-L1 thoracolumbar curve of 35º and a Risser 0. As a general rule we could say that below 50º the treatment is usually conservative, with two options: observation or use of a brace to slow down the evolution. In this case, taking into account the remaining growth (menarche 3 months ago and Risser 0) the most indicated would be to use a brace (Answer 2 correct)."}, "3": {"exist": false, "char_ranges": [], "word_ranges": [], "text": ""}, "4": {"exist": false, "char_ranges": [], "word_ranges": [], "text": ""}, "5": {"exist": false, "char_ranges": [], "word_ranges": [], "text": ""}}} +{"id": 146, "year": 2012, "question_id_specific": 231, "full_question": "A 7-month-old infant comes to the emergency department referred by his pediatrician for suspected Kawasaki disease. He reports a fever of up to 39.5ºC of 5 days evolution that does not subside despite treatment with amoxicillin prescribed 72 hours ago for suspected acute pharyngotonsillitis. During the last 24 hours he has had erythematous and pruritic exanthema of cephalocaudal evolution and during the last few days his parents have noticed a cough and intense conjunctivitis. No rhinorrhea was reported. Clinical examination showed subcrepitant in both lung bases without tachypnea or signs of respiratory distress. On oropharyngeal examination there are whitish spots in the jugal mucosa. What is the most probable diagnosis?", "full_answer": "The correct answer is 5. The clue is in the whitish lesions of the jugal mucosa corresponding to Koplik's spots, which are pathognomonic of measles, although as an observation I would say that they usually appear in the prodromal phase of the disease and are often no longer seen when the exanthema appears. In this case you are describing measles complicated by pneumonia. Very timely question at the present time.", "type": "PEDIATRICS", "options": {"1": "Kawasaki disease.", "2": "Scarlet fever.", "3": "Allergic reaction to amoxicillin.", "4": "Infectious mononucleosis.", "5": "Measles."}, "correct_option": 5, "explanations": {"1": {"exist": false, "char_ranges": [], "word_ranges": [], "text": ""}, "2": {"exist": false, "char_ranges": [], "word_ranges": [], "text": ""}, "3": {"exist": false, "char_ranges": [], "word_ranges": [], "text": ""}, "4": {"exist": false, "char_ranges": [], "word_ranges": [], "text": ""}, "5": {"exist": true, "char_ranges": [[25, 148]], "word_ranges": [[5, 25]], "text": "The clue is in the whitish lesions of the jugal mucosa corresponding to Koplik's spots, which are pathognomonic of measles,"}}} +{"id": 467, "year": 2020, "question_id_specific": 92, "full_question": "A woman presenting with difficulty walking after gynecologic surgery. She has mild thigh pain and leg failure on support. On examination she has weakness in hip flexion and knee extension, and dysesthesias in the anterior aspect of the thigh. What is the most probable diagnosis of suspicion:", "full_answer": "The femoral nerve (although rare) may be damaged in a cesarean section, hysterectomy or lower abdominal operation. This nerve injury produces hypoesthesia and weakness along its distribution. Femoroneuropathy is manifested by paralysis (25% of cases) of the quadriceps muscles, abolition of patellar reflexes and hypoesthesia of the anterior and inner thigh. Therefore, answer 1 is correct. The femoral cutaneous or lateral femoral cutaneous nerve (L2,L3) is only sensitive and is responsible for the sensitivity of the skin of the lateral and anterior aspect of the thigh up to the knee, so it would not explain the motor problem of gait and 2 is false. The obturator nerve (L2-L4) is a mixed nerve. At the motor level, it invests the obturator externus, pectineus, short approximator, the approximators (adductors) and gracilis. At the sensory level the capsule of the coxofemoral joint, medial aspect of the thigh above the knee. Therefore neither affects the sensitivity of the anterior aspect of the thigh nor the extension of the knee so that 3 is false. A lesion of the sciatic nerve at motor level affects to the flexion of the knee not to its extension and at distal level to this one (they do not count anything of distal clinic) and at sensitive level also it is distal in the external face of the leg and in the foot reason why 4 also is false.", "type": "ORTHOPEDIC SURGERY AND TRAUMATOLOGY", "options": {"1": "Femoral nerve neuropathy.", "2": "Meralgia paresthetica of the femoral cutaneous nerve.", "3": "Obturator nerve neuropathy.", "4": "Neuropathy of the sciatic nerve.", "5": NaN}, "correct_option": 1, "explanations": {"1": {"exist": true, "char_ranges": [[0, 390]], "word_ranges": [[0, 56]], "text": "The femoral nerve (although rare) may be damaged in a cesarean section, hysterectomy or lower abdominal operation. This nerve injury produces hypoesthesia and weakness along its distribution. Femoroneuropathy is manifested by paralysis (25% of cases) of the quadriceps muscles, abolition of patellar reflexes and hypoesthesia of the anterior and inner thigh. Therefore, answer 1 is correct."}, "2": {"exist": true, "char_ranges": [[391, 654]], "word_ranges": [[56, 104]], "text": "The femoral cutaneous or lateral femoral cutaneous nerve (L2,L3) is only sensitive and is responsible for the sensitivity of the skin of the lateral and anterior aspect of the thigh up to the knee, so it would not explain the motor problem of gait and 2 is false."}, "3": {"exist": true, "char_ranges": [[655, 1060]], "word_ranges": [[104, 170]], "text": "The obturator nerve (L2-L4) is a mixed nerve. At the motor level, it invests the obturator externus, pectineus, short approximator, the approximators (adductors) and gracilis. At the sensory level the capsule of the coxofemoral joint, medial aspect of the thigh above the knee. Therefore neither affects the sensitivity of the anterior aspect of the thigh nor the extension of the knee so that 3 is false."}, "4": {"exist": true, "char_ranges": [[1061, 1356]], "word_ranges": [[170, 230]], "text": "A lesion of the sciatic nerve at motor level affects to the flexion of the knee not to its extension and at distal level to this one (they do not count anything of distal clinic) and at sensitive level also it is distal in the external face of the leg and in the foot reason why 4 also is false."}, "5": {"exist": false, "char_ranges": [], "word_ranges": [], "text": ""}}} +{"id": 423, "year": 2018, "question_id_specific": 89, "full_question": "A 55-year-old woman, type 2 diabetic and obese, on treatment with metformin, with Hb A1c of 8%, has a history of repeated urinary tract infections. Which of the following therapeutic options to associate with metformin do you consider the LEAST appropriate?", "full_answer": "The most frequent side effects of the use of SGLT2 inhibitors are genitourinary tract infections. Since our patient presents with recurrent UTIs, the least indicated drug for the control of her diabetes would be option 4.", "type": "ENDOCRINOLOGY", "options": {"1": "DPP4 inhibitors.", "2": "GLP1 analogs.", "3": "Basal insulin.", "4": "SGLT2 inhibitors.", "5": NaN}, "correct_option": 4, "explanations": {"1": {"exist": false, "char_ranges": [], "word_ranges": [], "text": ""}, "2": {"exist": false, "char_ranges": [], "word_ranges": [], "text": ""}, "3": {"exist": false, "char_ranges": [], "word_ranges": [], "text": ""}, "4": {"exist": true, "char_ranges": [[0, 221]], "word_ranges": [[0, 36]], "text": "The most frequent side effects of the use of SGLT2 inhibitors are genitourinary tract infections. Since our patient presents with recurrent UTIs, the least indicated drug for the control of her diabetes would be option 4."}, "5": {"exist": false, "char_ranges": [], "word_ranges": [], "text": ""}}} +{"id": 449, "year": 2018, "question_id_specific": 151, "full_question": "A 13-year-old boy presents with subacute headache and diplopia. Neurologic examination shows vertical gaze palsy and MRI shows a contrast-enhancing lesion in the pineal region obstructing the aqueduct of Sylvius. The most likely diagnosis is:", "full_answer": "Germinal tumors are the most frequent in children in the pineal region. They usually capitate homogeneously and cause triventricular hydrocephalus and Parinaud. Glioblastomas usually occur in adults at the lobar level, medulloblastomas are the most frequent malignant tumors at the posterior fossa level in children (vermis and roof of the IV ventricle) but rare at the pineal level and meningiomas are rare in children and also in the pineal region.", "type": "NEUROSURGERY", "options": {"1": "Glioblastoma.", "2": "Medulloblastoma.", "3": "Germ cell tumor.", "4": "Meningioma.", "5": NaN}, "correct_option": 3, "explanations": {"1": {"exist": true, "char_ranges": [[161, 218]], "word_ranges": [[22, 31]], "text": "Glioblastomas usually occur in adults at the lobar level,"}, "2": {"exist": true, "char_ranges": [[219, 382]], "word_ranges": [[31, 58]], "text": "medulloblastomas are the most frequent malignant tumors at the posterior fossa level in children (vermis and roof of the IV ventricle) but rare at the pineal level"}, "3": {"exist": true, "char_ranges": [[0, 160]], "word_ranges": [[0, 22]], "text": "Germinal tumors are the most frequent in children in the pineal region. They usually capitate homogeneously and cause triventricular hydrocephalus and Parinaud."}, "4": {"exist": true, "char_ranges": [[387, 450]], "word_ranges": [[59, 70]], "text": "meningiomas are rare in children and also in the pineal region."}, "5": {"exist": false, "char_ranges": [], "word_ranges": [], "text": ""}}} +{"id": 433, "year": 2018, "question_id_specific": 118, "full_question": "Luis is a 25-year-old young man who underwent splenectomy after a bicycle accident 1 year ago. He has a dog that bit him 24 hours ago and has caused a small wound on his right hand. She comes to her health center (located 3 hours from the nearest hospital) with fever of 39°C, pain in the wound and general malaise. On examination, BP 100/60 mm Hg, HR 110 beats per minute, slight swelling in the wound without pus. Which of the following actions is most indicated at this time?", "full_answer": "This question has been complicated for us. The question does not have all the necessary information, but using what we have we can try to answer. The dog is domestic, known and can be monitored, so we would have to wait for the observation period to pass and prophylaxis is not recommended. At this point we will only have to worry about the possible bacteremia that may be occurring in a patient with the possibility of high risk due to asplenia. The treatment of choice in dog bite is amoxicillin/clavulanic acid but it is not available, so we can use another treatment such as clindamycin plus cephalosporin or clindamycin + moxifloxacin but it is not contemplated among the alternatives. In a first attempt we answered 4, but the correct answer is 3, because of the second part of the question; it is a high risk patient with incipient signs of sepsis so he should be sent to the hospital, here the key more than the antibiotic is in the position to take, in the second part of each answer. It could be challenged since they do not say in which community the bite occurred (if it was in Ceuta or Melilla the protocol may vary).", "type": "INFECTIOUS DISEASES AND MICROBIOLOGY", "options": {"1": "Send to hospital for rabies and tetanus vaccination and keep under observation.", "2": "Clean the wound and administer intramuscular nonspecific gamma globulin.", "3": "Give 400 mg of oral moxifloxacin and send to hospital.", "4": "Give clindamycin 600 mg orally every 8 hours and observation.", "5": NaN}, "correct_option": 3, "explanations": {"1": {"exist": false, "char_ranges": [], "word_ranges": [], "text": ""}, "2": {"exist": false, "char_ranges": [], "word_ranges": [], "text": ""}, "3": {"exist": true, "char_ranges": [[692, 994]], "word_ranges": [[119, 179]], "text": "In a first attempt we answered 4, but the correct answer is 3, because of the second part of the question; it is a high risk patient with incipient signs of sepsis so he should be sent to the hospital, here the key more than the antibiotic is in the position to take, in the second part of each answer."}, "4": {"exist": true, "char_ranges": [[692, 994]], "word_ranges": [[119, 179]], "text": "In a first attempt we answered 4, but the correct answer is 3, because of the second part of the question; it is a high risk patient with incipient signs of sepsis so he should be sent to the hospital, here the key more than the antibiotic is in the position to take, in the second part of each answer."}, "5": {"exist": false, "char_ranges": [], "word_ranges": [], "text": ""}}} +{"id": 337, "year": 2016, "question_id_specific": 31, "full_question": "20-year-old woman with ovarian tumor of 15 cm, solid-cystic, detected by ultrasound after presenting with unspecific abdominal symptoms. In the histopathological study of the corresponding specimen, teeth, hairs, areas of intestinal epithelium, areas of squamous epithelium (15%) and bronchial epithelium, as well as neuroectodermal and embryonic elements are found in several of the histological preparations. In reference to this case, point out the correct diagnosis:", "full_answer": "The answer is 3, mature cystic teratoma. This is due to the characteristics of the tumor: mature teratomas are constituted by adult tissues, coming from the three embryonic leaves, with a clear predominance of ectodermal elements (epidermis, pilosebaceous structures and neural tissue). Mature teratoma is a tumor of ovarian germ cells.", "type": "GYNECOLOGY AND OBSTETRICS", "options": {"1": "Teratocarcinoma.", "2": "Immature teratoma.", "3": "Mature cystic teratoma.", "4": "Dysgerminoma.", "5": NaN}, "correct_option": 3, "explanations": {"1": {"exist": false, "char_ranges": [], "word_ranges": [], "text": ""}, "2": {"exist": false, "char_ranges": [], "word_ranges": [], "text": ""}, "3": {"exist": true, "char_ranges": [[41, 286]], "word_ranges": [[7, 42]], "text": "This is due to the characteristics of the tumor: mature teratomas are constituted by adult tissues, coming from the three embryonic leaves, with a clear predominance of ectodermal elements (epidermis, pilosebaceous structures and neural tissue)."}, "4": {"exist": false, "char_ranges": [], "word_ranges": [], "text": ""}, "5": {"exist": false, "char_ranges": [], "word_ranges": [], "text": ""}}} +{"id": 296, "year": 2016, "question_id_specific": 17, "full_question": "A 47-year-old man, active smoker, with a history of drug abuse by parenteral route comes to the ED for a 2-hour evolution of acute onset pleuritic left costal pain and progressive dyspnea until becoming resting. In the previous 2 weeks he had presented febrile fever, general malaise and dry cough. On examination, the patient is tachypneic at 36 rpm, with supraclavicular pull, oxygen saturation by pulse oximetry of 81%, cyanosis, sweating, tachycardia at 135 beats per minute and BP of 75/47 mm Hg. The trachea is deviated to the right side and auscultation shows complete absence of vesicular murmur in the left hemithorax. HIV serology is positive and shows an ADC lymphocyte count of 176 cells/ml. What is the most likely diagnosis?", "full_answer": "Patient with risk factor for HIV acquisition, who comes to the emergency department for a subacute onset of 2 weeks of evolution (fever, malaise and dry cough) that worsens abruptly (pleuritic pain) and progressively (dyspnea). Physical examination suggests a pneumothorax (tachypnea, tugging, low saturation, cyanosis, diaphoresis, tachycardia, deviated trachea and absence of vesicular murmur). He has a positive HIV serology and a CD4 less than 200 cells/mL. The radiograph shows hyperclarity in the left lung field with absence of vessels between the lung border and the chest wall on the left side, displacement of the mediastinum and trachea to the right side, and a stump in the left pulmonary hilum corresponding to the collapsed lung. The suspicion of HIV is high and although it is pending confirmation, in the emergency it is valid to assume this option. That said, I am looking for pathologies that are associated with this picture, the rupture of a bulla although it is related to the appearance of abrupt symptoms does not go hand in hand with the other symptoms (you have to look for something that explains everything or almost everything), pneumonia although it is true that it can be related to many of the symptoms does not correlate with the image, Pulmonary TB could be manifested with all of the above and cause a pneumothorax, but here in a patient with a respiratory picture with more than 7 days of evolution that is complicated abruptly, in a patient with a high suspicion of HIV infection the clearest option goes for pneumonia due to P. jirovecii pneumonia. In patients with Pneumocystis, 10%, pneumatoceles appear which can sometimes open into the pleural space producing a spontaneous pneumothorax. As a reminder, the most frequent germs in pulmonary involvement in patients with HIV/AIDS infection are P. jiroveci (alone or associated with tuberculosis or other mycobacteria), S. pneumoniae, H. influenzae, S. aureus, cytomegalovirus, Legionella, staphylococci and gram-negative bacilli (Pseudomonas). Patients with P. jirovecii pneumonia have, in 90% of cases, a CD4 count < 250 cells/mL. Other agents that can cause pulmonary involvement less frequently are Rhodococcus equi, Nocardia, Aspergillus, Cryptococcus, Toxoplasma and Leishmania. Although the frequency of the different opportunistic infections has varied as a consequence of the administration of highly effective antiretroviral treatments, our patient would be in a setting where he has not yet received HAART.", "type": "INFECTIOUS DISEASES", "options": {"1": "Subpleural emphysematous bulla rupture.", "2": "Community-acquired necrotizing bacterial pneumonia.", "3": "Pulmonary tuberculosis.", "4": "Pneumocystis jiroveci pneumonia.", "5": NaN}, "correct_option": 4, "explanations": {"1": {"exist": true, "char_ranges": [[945, 1156]], "word_ranges": [[150, 187]], "text": "the rupture of a bulla although it is related to the appearance of abrupt symptoms does not go hand in hand with the other symptoms (you have to look for something that explains everything or almost everything),"}, "2": {"exist": true, "char_ranges": [[1157, 1268]], "word_ranges": [[187, 208]], "text": "pneumonia although it is true that it can be related to many of the symptoms does not correlate with the image,"}, "3": {"exist": true, "char_ranges": [[1359, 1585]], "word_ranges": [[224, 263]], "text": "in a patient with a respiratory picture with more than 7 days of evolution that is complicated abruptly, in a patient with a high suspicion of HIV infection the clearest option goes for pneumonia due to P. jirovecii pneumonia."}, "4": {"exist": true, "char_ranges": [[1359, 1585]], "word_ranges": [[224, 263]], "text": "in a patient with a respiratory picture with more than 7 days of evolution that is complicated abruptly, in a patient with a high suspicion of HIV infection the clearest option goes for pneumonia due to P. jirovecii pneumonia."}, "5": {"exist": false, "char_ranges": [], "word_ranges": [], "text": ""}}} +{"id": 76, "year": 2012, "question_id_specific": 87, "full_question": "76-year-old woman with obesity and type 2 diabetes mellitus on metformin. She comes to the emergency room because for 3 days she presents fever of 38ºC, polyuria, dysuria, intense thirst and progressive decrease in the level of consciousness. Physical examination showed signs of dehydration, BP 95/54 mm Hg and no signs of neurological localization. The CBC shows leukocytosis, creatinine of 1.8 mg/dL and blood glucose of 855 mg/dL. What is the most likely diagnosis?", "full_answer": "The question relates the case of a type 2 diabetic patient with hyperglycemia and decreased level of consciousness without neurological focality in the context of a urinary tract infection and who has developed acute renal failure of probable prerenal origin. Option 2 is immediately ruled out because the clinical picture is not one of heart failure. Option 3 is also discarded because it does not fit with the clinical picture and in addition HSD would give neurological symptoms with locality. Option 4 is discarded because obstructive renal failure does not give polyuria, dysuria or the rest of the clinical picture. Option 5 is less likely because diabetic ketoacidosis is more typical of type 1 DM. Therefore the correct option is 1.", "type": "ANESTHESIOLOGY, CRITICAL CARE AND EMERGENCIES", "options": {"1": "Hyperglycemic hyperosmolar non-ketotic hyperglycemic coma.", "2": "Cardiac failure.", "3": "Subdural hematoma.", "4": "Obstructive renal failure.", "5": "Diabetic ketoacidosis with coma."}, "correct_option": 1, "explanations": {"1": {"exist": false, "char_ranges": [], "word_ranges": [], "text": ""}, "2": {"exist": true, "char_ranges": [[260, 351]], "word_ranges": [[40, 56]], "text": "Option 2 is immediately ruled out because the clinical picture is not one of heart failure."}, "3": {"exist": true, "char_ranges": [[352, 496]], "word_ranges": [[56, 80]], "text": "Option 3 is also discarded because it does not fit with the clinical picture and in addition HSD would give neurological symptoms with locality."}, "4": {"exist": true, "char_ranges": [[497, 621]], "word_ranges": [[80, 100]], "text": "Option 4 is discarded because obstructive renal failure does not give polyuria, dysuria or the rest of the clinical picture."}, "5": {"exist": true, "char_ranges": [[622, 705]], "word_ranges": [[100, 115]], "text": "Option 5 is less likely because diabetic ketoacidosis is more typical of type 1 DM."}}} +{"id": 159, "year": 2012, "question_id_specific": 111, "full_question": "A 45-year-old male patient with a history of uric lithiasis with repeated expulsive nephritic colic for 25 years, comes to the ED referring palpitations and intense right lumbar pain since 2 hours before. Urinalysis showed a pH of 5.5 and leukocyturia without proteinuria. The ECG confirms a previously unknown atrial fibrillation. Blood biochemistry shows a creatinine of 0.9mg/dl, a calcium of 11mg/dl and an LDH of 950 U/l. What is the most useful diagnostic maneuver to perform?", "full_answer": "In this question I have more doubts as to which is the correct answer, since we must interpret which is the most useful test, with which we can obtain more performance. However, this situation also depends on the radiology services of each center and their preferences. I will focus on the urological part, leaving aside the rest of the symptomatology. - Abdominal X-ray is the first imaging study to be performed because it is fast, simple and practically innocuous (except in children and pregnant women). 90% of the stones are radiopaque and therefore visible, but the sensitivity of this test, used in isolation, decreases to 45-59%. It is a mandatory test in any suspicion of colic pain. However, it has a lower yield than the others. Although the urological history speaks of uric lithiasis (radiotransparent), with an acid pH that reaffirms the diagnosis of uric origin, we can never rule out calcium oxalate lithiasis that can appear synchronously. - IVUS seems to me to be one of the best performing options. It will give us diagnostic clarity of obstruction, its magnitude and its renal repercussions. It has long been considered the examination of choice for the study of patients with renal lithiasis. It is a test with high sensitivity and specificity (87-90% and 94-100%, respectively), relatively innocuous, and available in all hospitals. It informs us of the number, size, shape and location of the stone, as well as the functional status of the affected kidney. - Contrast-enhanced CT seems to me to be the most efficient option. It serves to discover lithiasis that other tests cannot find and gives us a global image of the rest of the structures. Great performance but also more expensive, more time, more radiation. It has been confirmed as a high sensitivity and specificity scan, surpassing previous tests (94-100% and 92-100%, respectively, for the detection of ureteral calculi) and with its numerous advantages it has become the reference test and is expected to replace IVUS and ultrasound worldwide. It performs a helical scan with 5 mm slices allowing the detection of stones up to 2 mm. With the administration of contrast, it allows the evaluation of renal function. - Ultrasound is a fast, cheap and innocuous test, but very radiologist-dependent. It gives great renal information on obstructive repercussions, but less quality in terms of finding a lithiasis. Ultrasonography only detects calculi larger than 4 mm, located in the pyelo-ureteral junction or the uretero-vesical junction, while the lumbar and pelvic ureter is not very accessible due to the interposition of the intestinal loops. Therefore, ultrasound performed in isolation is not very sensitive (20-45%) for the detection of calculi, but associated with simple abdominal radiography increases its sensitivity and specificity. - Doppler echocardiography has no use in urological diagnosis. My answer would be CT with ev contrast and excretory phase (answer (c)), in terms of performance, sensitivity and specificity. There are radiology services more skilled or predisposed to one or the other. Without forgetting that abdominal X-ray should always be done in the suspicion of colic.", "type": "UROLOGY", "options": {"1": "Simple abdominal X-ray.", "2": "IVU.", "3": "CT with contrast.", "4": "Abdominal echography.", "5": "Doppler echocardiography."}, "correct_option": 3, "explanations": {"1": {"exist": false, "char_ranges": [], "word_ranges": [], "text": ""}, "2": {"exist": true, "char_ranges": [[959, 1213]], "word_ranges": [[159, 202]], "text": "IVUS seems to me to be one of the best performing options. It will give us diagnostic clarity of obstruction, its magnitude and its renal repercussions. It has long been considered the examination of choice for the study of patients with renal lithiasis."}, "3": {"exist": true, "char_ranges": [[1482, 1667]], "word_ranges": [[246, 279]], "text": "Contrast-enhanced CT seems to me to be the most efficient option. It serves to discover lithiasis that other tests cannot find and gives us a global image of the rest of the structures."}, "4": {"exist": true, "char_ranges": [[2201, 2393]], "word_ranges": [[364, 392]], "text": "Ultrasound is a fast, cheap and innocuous test, but very radiologist-dependent. It gives great renal information on obstructive repercussions, but less quality in terms of finding a lithiasis."}, "5": {"exist": true, "char_ranges": [[2829, 2889]], "word_ranges": [[454, 462]], "text": "Doppler echocardiography has no use in urological diagnosis."}}} +{"id": 52, "year": 2011, "question_id_specific": 144, "full_question": "A 36-year-old patient from another culture who has been living in Spain for the last 4 years presents at a National Health Service office. He reports presenting with anxious symptoms and depressive mood for the last 10 months. This condition moderately interferes with her daily activities. She has no previous psychiatric history. This situation is the result of the death, in a car accident, of an older brother with whom he was very close. Indicate which of the following diagnoses is appropriate:", "full_answer": "The relative mildness of the symptoms, which do not seriously interfere with daily activities, excludes in principle the diagnosis of major depression. There is no evidence of hypomanic phases to justify a bipolar II disorder. Dysthymia requires a duration of at least two years. Cyclothymia is a very rare diagnosis nowadays (because, as Castilla del Pino said in an interview, what used to be cyclothymics are now called bipolar, contributing to one of the multiple psychiatric epidemics of our time), which consists of oscillations between the depressive pole and the manifest, mild. But there are no data on such oscillations. The correct answer is, in all probability, 3. The adaptive disorder appears within 3 months after the stressful event and does not last more than 6 months if the stressful event or its consequences have ceased (which is not the case because the brother of the imaginary patient is still dead). Mild anxious-depressive symptoms fit such a diagnosis. One more example of how to conceptualize as illness what are facts (albeit painful) of life. Surely the imaginary patient takes some antidepressant because maybe something helps (1).", "type": "PSYCHIATRY", "options": {"1": "Major depression.", "2": "Bipolar II disorder.", "3": "Adjustment disorder.", "4": "Dysthymia.", "5": "Cyclothymia."}, "correct_option": 3, "explanations": {"1": {"exist": true, "char_ranges": [[0, 151]], "word_ranges": [[0, 22]], "text": "The relative mildness of the symptoms, which do not seriously interfere with daily activities, excludes in principle the diagnosis of major depression."}, "2": {"exist": true, "char_ranges": [[152, 226]], "word_ranges": [[22, 35]], "text": "There is no evidence of hypomanic phases to justify a bipolar II disorder."}, "3": {"exist": true, "char_ranges": [[677, 979]], "word_ranges": [[108, 158]], "text": "The adaptive disorder appears within 3 months after the stressful event and does not last more than 6 months if the stressful event or its consequences have ceased (which is not the case because the brother of the imaginary patient is still dead). Mild anxious-depressive symptoms fit such a diagnosis."}, "4": {"exist": true, "char_ranges": [[227, 279]], "word_ranges": [[35, 44]], "text": "Dysthymia requires a duration of at least two years."}, "5": {"exist": true, "char_ranges": [[280, 630]], "word_ranges": [[44, 100]], "text": "Cyclothymia is a very rare diagnosis nowadays (because, as Castilla del Pino said in an interview, what used to be cyclothymics are now called bipolar, contributing to one of the multiple psychiatric epidemics of our time), which consists of oscillations between the depressive pole and the manifest, mild. But there are no data on such oscillations."}}} +{"id": 297, "year": 2016, "question_id_specific": 50, "full_question": "A 58-year-old man reported a weeks-long history of progressive, mildly painful skin lesions on his left arm. It had started as an erythematous lesion on his left thumb. He had visible reddish striae as connecting lines between the lesions. The patient had no fever or other general symptoms. He had been working in his garden but did not remember any injuries. The etiologic diagnosis was made by culture of a skin biopsy. What is the most likely causative agent of this process?", "full_answer": "The patient carries out an activity in contact with soil and plants (which are not provided free of charge). The symptoms are chronic, which started distally (left thumb) and are affecting the ipsilateral arm (reddish striae). There were no systemic symptoms. They also comment that the diagnosis was made on the basis of a skin biopsy. Of the microorganisms discussed, Microsporum gypseum does not usually manifest dissemination of infection of a lymphangitic aspect, in general keratinophilic fungi (dermatophytes) do not have any ability to invade deep tissues, so we ruled out this option. Staphylococcus aureus infection can affect superficial and deep tissue layers, they can spread following a lymphangitic path, but it is not usual such a long evolution, it is usually acute (days) and usually accompanied by systemic symptoms, so we discard this option. Mycobacterium marinum (and not Mycobacterum marinum) is a non-tuberculous mycobacterium characterized by being environmental, opportunistic and photochromogenic. It has a slow growth, from 2 to 8 weeks, at a temperature ranging from 30 to 37 °C. It affects different species of fish in cold, warm, fresh or salt water, especially stagnant waters of fish tanks and pools without chlorine. Infection in humans occurs by direct contact with fish or contaminated water in the presence of a loss of continuity in the skin of the host. Our patient has no history of exposure to fish or water (they don't say), so considering that usually the diagnosis is based on a high index of suspicion and specifically asking for this history, our case is far from this possibility. The Sporothrix schenckii complex is the causative agent of Sporotrichosis, this dimorphic fungus is located in the soil, the complex is formed by 5 species, S. schenckii sensu strictu, S. brasiliensis and S. globosa, capable of causing human and animal disease; the other two are S. mexicana and S. albicana not associated with any disease. Patients are usually infected by exposure to soil by means of a sharp object (thorns, stems, animal claws, etc.) and have a slow evolution, affect deep tissues and their dissemination typically follows a pathway as described in the question.", "type": "INFECTIOUS DISEASES", "options": {"1": "Dermatophytosis by Microsporum gypseum.", "2": "Cutaneous infection by Staphylococcus aureus.", "3": "Sporotrichosis.", "4": "Cutaneous infection by Mycobacterium marinum.", "5": NaN}, "correct_option": 3, "explanations": {"1": {"exist": true, "char_ranges": [[370, 564]], "word_ranges": [[60, 87]], "text": "Microsporum gypseum does not usually manifest dissemination of infection of a lymphangitic aspect, in general keratinophilic fungi (dermatophytes) do not have any ability to invade deep tissues,"}, "2": {"exist": true, "char_ranges": [[594, 835]], "word_ranges": [[93, 130]], "text": "Staphylococcus aureus infection can affect superficial and deep tissue layers, they can spread following a lymphangitic path, but it is not usual such a long evolution, it is usually acute (days) and usually accompanied by systemic symptoms,"}, "3": {"exist": true, "char_ranges": [[1969, 2210]], "word_ranges": [[316, 355]], "text": "Patients are usually infected by exposure to soil by means of a sharp object (thorns, stems, animal claws, etc.) and have a slow evolution, affect deep tissues and their dissemination typically follows a pathway as described in the question."}, "4": {"exist": true, "char_ranges": [[1393, 1627]], "word_ranges": [[220, 261]], "text": "Our patient has no history of exposure to fish or water (they don't say), so considering that usually the diagnosis is based on a high index of suspicion and specifically asking for this history, our case is far from this possibility."}, "5": {"exist": false, "char_ranges": [], "word_ranges": [], "text": ""}}} +{"id": 74, "year": 2012, "question_id_specific": 63, "full_question": "A 67-year-old woman with a history of congestive heart failure (treated with ramipril and furosemide) and type 2 diabetes (treated with insulin) comes to the emergency room for dyspnea. Blood gas analysis performed while breathing room air shows pH 7.45, paO2 56, PaCO2 30 mm Hg, HCO3 26 mm Hg. What gasometric alteration does the patient present?", "full_answer": "I found this question tricky. The separate findings are: normal pH but with alkalosis tendency, acute respiratory failure, hypocapnia and normal bicarbonate. The most appropriate answer seems to me to be 1, since the patient does not present a metabolic alkalosis (bicarbonate would be higher), much less an acidosis (bicarbonate would be low). The alkalosis is not mixed either, but is mainly respiratory. The patient is probably hyperventilating because of her respiratory failure, and the kidney which always compensates later is starting to retain HCO3 to compensate and maintain pH. Therefore I believe the correct answer is 1.", "type": "ANESTHESIOLOGY, CRITICAL CARE AND EMERGENCIES", "options": {"1": "Hypoxemic respiratory failure with compensated respiratory alkalosis.", "2": "Acute metabolic alkalosis with hypoxemic and hypocapnic respiratory failure.", "3": "Chronic normocapnic respiratory insufficiency, without alteration of acid-base balance.", "4": "Chronic hypoxemic respiratory failure with compensated metabolic acidosis.", "5": "Hypoxemic respiratory failure with compensated mixed alkalosis."}, "correct_option": 1, "explanations": {"1": {"exist": true, "char_ranges": [[158, 587]], "word_ranges": [[22, 90]], "text": "The most appropriate answer seems to me to be 1, since the patient does not present a metabolic alkalosis (bicarbonate would be higher), much less an acidosis (bicarbonate would be low). The alkalosis is not mixed either, but is mainly respiratory. The patient is probably hyperventilating because of her respiratory failure, and the kidney which always compensates later is starting to retain HCO3 to compensate and maintain pH."}, "2": {"exist": false, "char_ranges": [], "word_ranges": [], "text": ""}, "3": {"exist": false, "char_ranges": [], "word_ranges": [], "text": ""}, "4": {"exist": false, "char_ranges": [], "word_ranges": [], "text": ""}, "5": {"exist": false, "char_ranges": [], "word_ranges": [], "text": ""}}} +{"id": 288, "year": 2016, "question_id_specific": 227, "full_question": "A 76-year-old patient with hypertension and diabetes mellitus comes to the emergency department because he has been experiencing palpitations and decreased capacity to exert himself for 72 hours. On arrival, atrial fibrillation with ventricular response of around 120 bpm was documented. Which of the following options is FALSE?", "full_answer": "Since it is the first episode of atrial fibrillation, it is the ideal candidate for catheter ablation.", "type": "CARDIOLOGY AND VASCULAR SURGERY", "options": {"1": "This patient should be orally anticoagulated for life, unless contraindicated.", "2": "If we decide to perform cardioversion on arrival at the ED, it would be necessary to perform transesophageal echocardiography beforehand.", "3": "Being the first episode of atrial fibrillation, it is the ideal candidate for catheter ablation.", "4": "Beta-blockers may be used to slow the heart rate.", "5": NaN}, "correct_option": 3, "explanations": {"1": {"exist": false, "char_ranges": [], "word_ranges": [], "text": ""}, "2": {"exist": false, "char_ranges": [], "word_ranges": [], "text": ""}, "3": {"exist": true, "char_ranges": [[0, 102]], "word_ranges": [[0, 17]], "text": "Since it is the first episode of atrial fibrillation, it is the ideal candidate for catheter ablation."}, "4": {"exist": false, "char_ranges": [], "word_ranges": [], "text": ""}, "5": {"exist": false, "char_ranges": [], "word_ranges": [], "text": ""}}} +{"id": 460, "year": 2018, "question_id_specific": 174, "full_question": "A 60-year-old, insulin-dependent diabetic woman presents with right-sided, nocturnal-predominant omalgia of several weeks' duration. She does not report any trauma. Physical examination shows active and passive limitation of all the arcs of movement of the shoulder. Which clinical picture do you suspect as the first diagnostic possibility?", "full_answer": "Everything points to adhesive capsulitis, as you say. The age, the sex, the clinic... the DM....", "type": "TRAUMATOLOGY AND ORTHOPEDICS", "options": {"1": "A malignant tumor located in the proximal epiphysis of the humerus.", "2": "Septic arthritis of the shoulder.", "3": "Adhesive capsulitis.", "4": "A degenerative acromioclavicular arthrosis process.", "5": NaN}, "correct_option": 3, "explanations": {"1": {"exist": false, "char_ranges": [], "word_ranges": [], "text": ""}, "2": {"exist": false, "char_ranges": [], "word_ranges": [], "text": ""}, "3": {"exist": true, "char_ranges": [[0, 96]], "word_ranges": [[0, 16]], "text": "Everything points to adhesive capsulitis, as you say. The age, the sex, the clinic... the DM...."}, "4": {"exist": false, "char_ranges": [], "word_ranges": [], "text": ""}, "5": {"exist": false, "char_ranges": [], "word_ranges": [], "text": ""}}} +{"id": 333, "year": 2016, "question_id_specific": 90, "full_question": "A 35-year-old woman consults for a right thyroid nodule detected incidentally one morning when she observed in the mirror a lump on the anterior aspect of the neck. After the pertinent studies, it was decided to operate on the patient, performing a total thyroidectomy with emptying of the central lymph node component. The pathologist's report was that the 2.3 cm thyroid nodule was entirely occupied by a papillary thyroid carcinoma, tall cell variant, without vascular infiltration but with capsular infiltration. Thyroglobulin concentrations 24 h after thyroidectomy are 14 ng/mL. What would be the next step you would take in this patient?", "full_answer": "The verification of the presence of thyroglobulin in blood and the development of techniques by which it can be determined have provided a new procedure for monitoring patients with papillary or follicular thyroid carcinoma. Thyroglobulin is synthesized in the follicular cells and then secreted into the circulation, therefore, after total thyroidectomy it should disappear, if this does not occur we must think about the existence of remnants of metastatic tissue.", "type": "ENDOCRINOLOGY", "options": {"1": "Refer patient back to surgeon for right laterocervical lymph node stripping.", "2": "Initiate treatment with a dose of TSH-suppressing levothyroxine and schedule for review 6 months later with a new analytical determination of thyroglobulin and cervical ultrasound.", "3": "Request a PET-CT scan for lymph node involvement.", "4": "Delay initiation of TSH replacement-suppressive treatment with levothyroxine and refer the patient to the Nuclear Medicine service for administration of an ablative dose of 100 mCi of I131.", "5": NaN}, "correct_option": 4, "explanations": {"1": {"exist": false, "char_ranges": [], "word_ranges": [], "text": ""}, "2": {"exist": false, "char_ranges": [], "word_ranges": [], "text": ""}, "3": {"exist": false, "char_ranges": [], "word_ranges": [], "text": ""}, "4": {"exist": true, "char_ranges": [[225, 466]], "word_ranges": [[34, 70]], "text": "Thyroglobulin is synthesized in the follicular cells and then secreted into the circulation, therefore, after total thyroidectomy it should disappear, if this does not occur we must think about the existence of remnants of metastatic tissue."}, "5": {"exist": false, "char_ranges": [], "word_ranges": [], "text": ""}}} +{"id": 185, "year": 2013, "question_id_specific": 66, "full_question": "A 74-year-old patient with bowel resection following mesenteric thrombosis comes to consult for fatigue and bone pain. He had suffered a Colles' fracture three months earlier. Laboratory tests showed anemia of 9.5 g/dl, microcytic, albumin 3.5 g/di, calcium 7.5 mg/dl, phosphate 2.0 mg/dl and alkaline phosphatase 224 IU (normal up to 120 IU). Radiographs of the femur show radiolucent bands perpendicular to the cortex on the inner side of the upper femur. What is the most likely diagnosis?", "full_answer": "Although this one had not been assigned to me, I am answering it as well since it deals with an endocrinology topic: vitamin D. In this case there are important data such as the history of intestinal resection, hypocalcemia and hypophosphoremoia; this should lead us to think in a vitamin D deficit due to malabsorption and therefore, in an osteomalacia. The rest of the data: previous fracture, pain and the X-ray support the diagnosis.", "type": "ENDOCRINOLOGY", "options": {"1": "Osteoporosis.", "2": "Paget's disease.", "3": "Osteitis fibrosa.", "4": "Prostate cancer metastasis.", "5": "Osteomalacia."}, "correct_option": 5, "explanations": {"1": {"exist": false, "char_ranges": [], "word_ranges": [], "text": ""}, "2": {"exist": false, "char_ranges": [], "word_ranges": [], "text": ""}, "3": {"exist": false, "char_ranges": [], "word_ranges": [], "text": ""}, "4": {"exist": false, "char_ranges": [], "word_ranges": [], "text": ""}, "5": {"exist": true, "char_ranges": [[128, 354]], "word_ranges": [[24, 60]], "text": "In this case there are important data such as the history of intestinal resection, hypocalcemia and hypophosphoremoia; this should lead us to think in a vitamin D deficit due to malabsorption and therefore, in an osteomalacia."}}} +{"id": 107, "year": 2012, "question_id_specific": 206, "full_question": "An 8-year-old girl (index case) is clinically diagnosed as having neurofibromatosis type 1 (NF1) or Von Recklinghausen's disease with multiple neurofibromas, café-au-lait spots, and Lisch nodules. Her father (not diagnosed with NF1) died in a traffic accident at the age of 38 years. On examination, the mother presents two café-au-lait spots and attends the genetic counseling consultation with her new partner where a preimplantation genetic diagnosis (PGD) is proposed. Is PGD indicated in this case?", "full_answer": "The first step is to detect the mutation and once detected, if the mother is a carrier, to proceed with PGD.", "type": "GENETICS AND IMMUNOLOGY", "options": {"1": "Yes, as the mother has 2 café-au-lait spots, she is a carrier and PGD is indicated with these data.", "2": "It is indicated after detection of the causative mutation in the index case and eventually in his mother.", "3": "It is not indicated because NF1 responds to mutations in the neurofibromin gene (17q11.2), with recessive inheritance.", "4": "No, two café-au-lait spots are not diagnostic and your new partner is very unlikely to be a carrier (NF1 is a rare disease).", "5": "With these data, PGD is indicated, consisting of selecting embryos in vitro, to implant in the maternal uterus those without the mutation."}, "correct_option": 2, "explanations": {"1": {"exist": false, "char_ranges": [], "word_ranges": [], "text": ""}, "2": {"exist": true, "char_ranges": [[0, 108]], "word_ranges": [[0, 21]], "text": "The first step is to detect the mutation and once detected, if the mother is a carrier, to proceed with PGD."}, "3": {"exist": false, "char_ranges": [], "word_ranges": [], "text": ""}, "4": {"exist": false, "char_ranges": [], "word_ranges": [], "text": ""}, "5": {"exist": false, "char_ranges": [], "word_ranges": [], "text": ""}}} +{"id": 114, "year": 2012, "question_id_specific": 96, "full_question": "A 43-year-old woman consults her primary care physician for fatigue, gingivorrhea and petechiae. Laboratory tests showed anemia of 8 g/dL, thrombopenia of 4000/microliter and leukopenia of 1200/microliter with absolute neutropenia. The coagulation study showed APTT elongation (43″), decreased prothrombin activity (55%), hypofibrinogenemia (98 mg/dl) and presence of high concentration of D-Dimer and fibrin monomers. She is referred for urgent hematologic workup and a bone marrow aspirate is performed showing massive infiltration by immature elements with cleaved nuclei and Auer rods in the cytoplasm. What is the most likely diagnosis in this patient?", "full_answer": "After reading such a long statement, running out of breath, going over it and getting overwhelmed, it is time to take a look at it and grasp at something very characteristic. A woman with pancytopenia. A bone marrow showing massive infiltration by immature elements that are proliferating non-stop...with Auer's canes? Acute promyelocytic leukemia. Those rods are the fusion of primary granules of developing myelocytes. Robin&Cotran say so on page 696 of their book \"General Pathology\" [1]. It is also said by Carr and Rodak in their book \"Atlas of clinical hematology\" on page 157 [2].", "type": "HEMATOLOGY", "options": {"1": "Chronic myeloid leukemia.", "2": "Acute promyelocytic leukemia.", "3": "Acute monoblastic leukemia.", "4": "Chronic lymphoid leukemia.", "5": "Acute lymphoid leukemia."}, "correct_option": 2, "explanations": {"1": {"exist": false, "char_ranges": [], "word_ranges": [], "text": ""}, "2": {"exist": true, "char_ranges": [[175, 420]], "word_ranges": [[31, 64]], "text": "A woman with pancytopenia. A bone marrow showing massive infiltration by immature elements that are proliferating non-stop...with Auer's canes? Acute promyelocytic leukemia. Those rods are the fusion of primary granules of developing myelocytes."}, "3": {"exist": false, "char_ranges": [], "word_ranges": [], "text": ""}, "4": {"exist": false, "char_ranges": [], "word_ranges": [], "text": ""}, "5": {"exist": false, "char_ranges": [], "word_ranges": [], "text": ""}}} +{"id": 61, "year": 2011, "question_id_specific": 92, "full_question": "A 30-year-old man suffers fall from a standing height of 2 meters. On arrival at the ER, he is cyo, catheterized with clear urine. He shows swelling of the left heel, as well as mild tingling in the anterior aspect of the right thigh and scrotum, and dysesthesia in both feet. The patient says goodbye to the stretcher-bearer with a handshake, with a simultaneous pat on the shoulder. Rotation of the MMII on the stretcher grasped by the ankles is painless and compression of the pelvis is asymptomatic. Next:", "full_answer": "From the symptoms reported, tingling in the anterior thigh, scrotum and dysesthesia in the feet, there seems to be a vertebral involvement at L2-3 and sacral roots, so the correct thing to do would be to perform a radiological examination without mobilizing the patient. But it is necessary to mobilize him to explore him, obviously, en bloc. The cervical spine seems to be undamaged because the patient mobilizes both arms adequately, without paresthesia.", "type": "ANESTHESIOLOGY AND CRITICAL CARE", "options": {"1": "We will sit him on the stretcher to be able to explore the cervical spine.", "2": "We will refer to Rx to perform lower extremities study.", "3": "Palpate the abdomen and rotate it en bloc to palpate the thoracolumbar spine.", "4": "Place a cervical collar and refer to radiology for cervical spine and lower extremity imaging.", "5": "We will request an abdominal echo."}, "correct_option": 3, "explanations": {"1": {"exist": false, "char_ranges": [], "word_ranges": [], "text": ""}, "2": {"exist": false, "char_ranges": [], "word_ranges": [], "text": ""}, "3": {"exist": true, "char_ranges": [[275, 456]], "word_ranges": [[45, 73]], "text": "it is necessary to mobilize him to explore him, obviously, en bloc. The cervical spine seems to be undamaged because the patient mobilizes both arms adequately, without paresthesia."}, "4": {"exist": false, "char_ranges": [], "word_ranges": [], "text": ""}, "5": {"exist": false, "char_ranges": [], "word_ranges": [], "text": ""}}} +{"id": 231, "year": 2014, "question_id_specific": 180, "full_question": "A 1-month-old infant comes to the hospital emergency department referred by his pediatrician for jaundice. He reports jaundice for 10 days and it has been increasing. The blood test showed a total bilirubin of 7mg/dl with an indirect bilirubin of 1.5mg/dl. The most probable cause, among the following, of this jaundice is:", "full_answer": "The increase in direct bilirubin immediately rules out the rest of the options where the increase in total bilirubin would be at the expense of indirect bilirubin. Bile duct atresia is the main indication for infant liver transplantation.", "type": "PEDIATRICS", "options": {"1": "Bile duct atresia.", "2": "Breastfeeding jaundice.", "3": "Late onset 0-A isoimmunization.", "4": "Gilbert's disease.", "5": "Hereditary spherocytosis."}, "correct_option": 1, "explanations": {"1": {"exist": true, "char_ranges": [[164, 238]], "word_ranges": [[27, 38]], "text": "Bile duct atresia is the main indication for infant liver transplantation."}, "2": {"exist": true, "char_ranges": [[0, 163]], "word_ranges": [[0, 27]], "text": "The increase in direct bilirubin immediately rules out the rest of the options where the increase in total bilirubin would be at the expense of indirect bilirubin."}, "3": {"exist": true, "char_ranges": [[0, 163]], "word_ranges": [[0, 27]], "text": "The increase in direct bilirubin immediately rules out the rest of the options where the increase in total bilirubin would be at the expense of indirect bilirubin."}, "4": {"exist": true, "char_ranges": [[0, 163]], "word_ranges": [[0, 27]], "text": "The increase in direct bilirubin immediately rules out the rest of the options where the increase in total bilirubin would be at the expense of indirect bilirubin."}, "5": {"exist": true, "char_ranges": [[0, 163]], "word_ranges": [[0, 27]], "text": "The increase in direct bilirubin immediately rules out the rest of the options where the increase in total bilirubin would be at the expense of indirect bilirubin."}}} +{"id": 87, "year": 2012, "question_id_specific": 54, "full_question": "An 81-year-old patient consults for an episode of syncope. He reports dyspnea on exertion for a year. On cardiac auscultation there is a 3/6 systolic murmur on the left sternal border radiating to the carotids and tip. Which pathology seems most likely?", "full_answer": "Very easy question. They are describing the \"portrait\" of a degenerative aortic stenosis... Elderly patient with clinical dyspnea and syncope, and systolic murmur radiating to carotids and tip, all typical of this disease.", "type": "CARDIOLOGY AND VASCULAR SURGERY", "options": {"1": "Third-degree atrioventricular block.", "2": "Degenerative aortic stenosis.", "3": "Mitral valve insufficiency.", "4": "Hypertrophic cardiomyopathy.", "5": "Dilated cardiomyopathy."}, "correct_option": 2, "explanations": {"1": {"exist": false, "char_ranges": [], "word_ranges": [], "text": ""}, "2": {"exist": true, "char_ranges": [[92, 222]], "word_ranges": [[13, 33]], "text": "Elderly patient with clinical dyspnea and syncope, and systolic murmur radiating to carotids and tip, all typical of this disease."}, "3": {"exist": false, "char_ranges": [], "word_ranges": [], "text": ""}, "4": {"exist": false, "char_ranges": [], "word_ranges": [], "text": ""}, "5": {"exist": false, "char_ranges": [], "word_ranges": [], "text": ""}}} +{"id": 102, "year": 2012, "question_id_specific": 87, "full_question": "A 76-year-old woman with obesity and type 2 diabetes mellitus on metformin. She comes to the emergency room because for 3 days she presents fever of 38ºC, pollakiuria, dysuria, intense thirst and progressive decrease in her level of consciousness. Physical examination showed signs of dehydration, blood pressure 95/54 mmHg and no signs of neurological focality. The CBC shows leukocytosis, creatinine of 1.8 mg/dL. What is the most likely diagnosis?", "full_answer": "Easy question of acute complications of diabetes mellitus.", "type": "ENDOCRINOLOGY", "options": {"1": "Non-ketotic hyperglycemic hyperosmolar hyperglycemic coma.", "2": "Cardiac insufficiency.", "3": "Subdural hematoma.", "4": "Renal insufficiency of obstructive cause.", "5": "Diabetic ketoacidosis with coma."}, "correct_option": 1, "explanations": {"1": {"exist": false, "char_ranges": [], "word_ranges": [], "text": ""}, "2": {"exist": false, "char_ranges": [], "word_ranges": [], "text": ""}, "3": {"exist": false, "char_ranges": [], "word_ranges": [], "text": ""}, "4": {"exist": false, "char_ranges": [], "word_ranges": [], "text": ""}, "5": {"exist": false, "char_ranges": [], "word_ranges": [], "text": ""}}} +{"id": 512, "year": 2021, "question_id_specific": 153, "full_question": "Patient diagnosed with acute diverticulitis, treated with amoxicillin/clavulanic acid. After 5 days he starts with fever and diarrhea. He was diagnosed with Clostridium difficile colitis, amoxicillin/clavulanic acid was discontinued and metronidazole was prescribed. After 4 days he has not responded to metronidazole, but is clinically stable. The next step of treatment is:", "full_answer": "The treatment of choice for Clostridioides difficile infection is vancomycin orally 125mg/6h for 10 days. Metronidazole has been relegated to situations where vancomycin is not available.", "type": "DIGESTIVE", "options": {"1": "Change metronidazole for oral vancomycin (125 mg, four times a day for 10 days).", "2": "Change metronidazole to piperacillin/ tazobactam (4 g piperacillin/0.5 g tazobactam every 8 hours).", "3": "Indicate a subtotal colectomy.", "4": "Loop ileostomy with antegrade polyethylene glycol lavage.", "5": NaN}, "correct_option": 1, "explanations": {"1": {"exist": true, "char_ranges": [[0, 105]], "word_ranges": [[0, 15]], "text": "The treatment of choice for Clostridioides difficile infection is vancomycin orally 125mg/6h for 10 days."}, "2": {"exist": false, "char_ranges": [], "word_ranges": [], "text": ""}, "3": {"exist": false, "char_ranges": [], "word_ranges": [], "text": ""}, "4": {"exist": false, "char_ranges": [], "word_ranges": [], "text": ""}, "5": {"exist": false, "char_ranges": [], "word_ranges": [], "text": ""}}} +{"id": 218, "year": 2014, "question_id_specific": 123, "full_question": "30-year-old man with no past history of interest. He comes for consultation due to the presence of small erythematous-violaceous lesions that on palpation appear to be raised in the pretibial region. The analytical study shows a complete blood count and coagulation study without alterations, and in the biochemistry, creatinine and ions are also within the normal range. The urinary sediment study shows hematuria, for which the patient had already been studied on other occasions, without obtaining a definitive diagnosis. Regarding the entity you suspect in this case, it is FALSE that:", "full_answer": "They are talking to us with high probability of a mesangial IgA glomerulonephritis or Berger's disease. Therefore, we are going to discard options one by one: 1: True. Serum IgA elevation is found in up to 50% of cases. 2: True. Mesangial IgA deposits are present in almost 100% of cases. 3: This option is false, because this glomerulonephritis is classically manifested with nephritic and not nephrotic syndrome (although in some rare cases proteinuria in nephrotic range does appear, but in the MIR they do not ask about these rare cases). 4: At the beginning this option generated doubts in me, but looking in the literature, it is true that the evolution to renal failure (according to last series) occurs in about 25% of the cases, so this option is true. 5: Skin biopsy, because it is easier to perform than renal biopsy, is the diagnostic technique of choice (the skin lesions that constitute Schonlein-Henoch purpura, so frequently associated with this entity and which the patient in the case presents, are biopsied).", "type": "NEPHROLOGY", "options": {"1": "In 20 to 50% of cases there is elevation of serum IgA concentration.", "2": "In the renal biopsy the mesangial deposits of IgA are characteristic.", "3": "It is frequent the existence of proteinuria in nephrotic range.", "4": "It is considered a benign entity since less than 1/3 of patients progress to renal failure.", "5": "The cutaneous biopsy allows to establish the diagnosis in up to half of the cases."}, "correct_option": 3, "explanations": {"1": {"exist": true, "char_ranges": [[159, 219]], "word_ranges": [[26, 39]], "text": "1: True. Serum IgA elevation is found in up to 50% of cases."}, "2": {"exist": true, "char_ranges": [[220, 288]], "word_ranges": [[39, 51]], "text": "2: True. Mesangial IgA deposits are present in almost 100% of cases."}, "3": {"exist": true, "char_ranges": [[289, 413]], "word_ranges": [[51, 68]], "text": "3: This option is false, because this glomerulonephritis is classically manifested with nephritic and not nephrotic syndrome"}, "4": {"exist": true, "char_ranges": [[543, 761]], "word_ranges": [[91, 131]], "text": "4: At the beginning this option generated doubts in me, but looking in the literature, it is true that the evolution to renal failure (according to last series) occurs in about 25% of the cases, so this option is true."}, "5": {"exist": true, "char_ranges": [[762, 1027]], "word_ranges": [[131, 172]], "text": "5: Skin biopsy, because it is easier to perform than renal biopsy, is the diagnostic technique of choice (the skin lesions that constitute Schonlein-Henoch purpura, so frequently associated with this entity and which the patient in the case presents, are biopsied)."}}} +{"id": 493, "year": 2020, "question_id_specific": 107, "full_question": "An 18-year-old patient comes to the emergency department with epistaxis of several days' evolution, with no personal or family history of interest. On examination he is afebrile, multiple ecchymoses are observed, no splenomegaly is palpable. Laboratory tests: leukocytes 7.2 x103/μL, Hb 12.3 g/dL, platelets 6.0 x103/μL. Thrombocytopenia is confirmed in the smear, where platelets of increased size are observed. Coagulation and biochemistry studies are normal. What do you consider the most probable diagnosis?", "full_answer": "If they wanted us to answer that it is a TTP (answer 1) they would probably associate anemia and in the smear they would surely tell us about schistocytes. Not to say that the patient's symptoms would probably be much more than epistaxis. The other two answers, you do not have to hesitate with them. Nothing points to that.", "type": "HEMATOLOGY", "options": {"1": "Thrombotic thrombocytopenic purpura.", "2": "Disseminated intravascular coagulation.", "3": "Thrombocytopenia induced by infection.", "4": "Primary immune thrombocytopenia.", "5": NaN}, "correct_option": 4, "explanations": {"1": {"exist": true, "char_ranges": [[0, 155]], "word_ranges": [[0, 29]], "text": "If they wanted us to answer that it is a TTP (answer 1) they would probably associate anemia and in the smear they would surely tell us about schistocytes."}, "2": {"exist": true, "char_ranges": [[239, 324]], "word_ranges": [[43, 59]], "text": "The other two answers, you do not have to hesitate with them. Nothing points to that."}, "3": {"exist": true, "char_ranges": [[239, 324]], "word_ranges": [[43, 59]], "text": "The other two answers, you do not have to hesitate with them. Nothing points to that."}, "4": {"exist": false, "char_ranges": [], "word_ranges": [], "text": ""}, "5": {"exist": false, "char_ranges": [], "word_ranges": [], "text": ""}}} +{"id": 566, "year": 2022, "question_id_specific": 149, "full_question": "A 67-year-old woman with chronic kidney disease secondary to diabetic nephropathy. She currently has a creatinine of 3.2 mg/dL with estimated glomerular filtration rate (CKD-EPI) 14 ml/min. Which of the following is true?", "full_answer": "A glomerular filtration rate of less than 15 ml/min is classified as stage 5 (G5), indicating that the renal disease is end-stage and the initiation of replacement therapy should be considered (option 2 true, option 1 false). In this situation, good glycemic control is not expected to reverse renal damage, although it may help to delay progression (false choice 3). These patients usually present stage A3 albuminuria (above 300 mg/24 h), and may reach the nephrotic range (false choice 4).", "type": "NEPHROLOGY", "options": {"1": "He presents stage 3 chronic kidney disease and should initiate phosphorus chelators.", "2": "He has stage 5 chronic kidney disease and renal replacement therapy should be considered.", "3": "Good glycemic control can reverse renal failure.", "4": "It is unlikely to present proteinuria higher than 500 mg/24 h.", "5": NaN}, "correct_option": 2, "explanations": {"1": {"exist": true, "char_ranges": [[0, 225]], "word_ranges": [[0, 37]], "text": "A glomerular filtration rate of less than 15 ml/min is classified as stage 5 (G5), indicating that the renal disease is end-stage and the initiation of replacement therapy should be considered (option 2 true, option 1 false)."}, "2": {"exist": true, "char_ranges": [[0, 225]], "word_ranges": [[0, 37]], "text": "A glomerular filtration rate of less than 15 ml/min is classified as stage 5 (G5), indicating that the renal disease is end-stage and the initiation of replacement therapy should be considered (option 2 true, option 1 false)."}, "3": {"exist": true, "char_ranges": [[226, 367]], "word_ranges": [[37, 60]], "text": "In this situation, good glycemic control is not expected to reverse renal damage, although it may help to delay progression (false choice 3)."}, "4": {"exist": true, "char_ranges": [[368, 492]], "word_ranges": [[60, 80]], "text": "These patients usually present stage A3 albuminuria (above 300 mg/24 h), and may reach the nephrotic range (false choice 4)."}, "5": {"exist": false, "char_ranges": [], "word_ranges": [], "text": ""}}} +{"id": 254, "year": 2014, "question_id_specific": 89, "full_question": "A 17-year-old boy, athletic and with no history of interest, comes to the doctor for occasional conjunctival jaundice. He denies drug intake and abdominal pain. Laboratory tests show normal ALT, AST, GGT and FA, with total bilirubin of 3.2 mg/dl and direct bilirubin of 0.4 mg/dl. She has no anemia and the liver is ultrasonographically normal. What is her diagnosis?", "full_answer": "An increase in indirect bilirubin less than 5 (intermittently, under stress) in a young healthy boy should make us think of Gilbert's syndrome. In Rotor syndrome, Dubin-Johnson syndrome and choledocholithiasis there is an increase in direct bilirubin. In the case of acute hepatitis there should be an elevation of transaminases.", "type": "DIGESTIVE TRACT", "options": {"1": "Rotor syndrome.", "2": "Choledocholithiasis.", "3": "Dubin-Johnson syndrome.", "4": "Acute hepatitis.", "5": "Gilbert's syndrome."}, "correct_option": 5, "explanations": {"1": {"exist": true, "char_ranges": [[144, 251]], "word_ranges": [[23, 37]], "text": "In Rotor syndrome, Dubin-Johnson syndrome and choledocholithiasis there is an increase in direct bilirubin."}, "2": {"exist": true, "char_ranges": [[144, 251]], "word_ranges": [[23, 37]], "text": "In Rotor syndrome, Dubin-Johnson syndrome and choledocholithiasis there is an increase in direct bilirubin."}, "3": {"exist": true, "char_ranges": [[144, 251]], "word_ranges": [[23, 37]], "text": "In Rotor syndrome, Dubin-Johnson syndrome and choledocholithiasis there is an increase in direct bilirubin."}, "4": {"exist": true, "char_ranges": [[252, 329]], "word_ranges": [[37, 50]], "text": "In the case of acute hepatitis there should be an elevation of transaminases."}, "5": {"exist": false, "char_ranges": [], "word_ranges": [], "text": ""}}} +{"id": 597, "year": 2022, "question_id_specific": 180, "full_question": "26-year-old woman diagnosed with systemic lupus erythematosus, on treatment with hydroxychloroquine, who consults for a feeling of generalized weakness that has progressively set in over the last 15 days. The physical examination reveals cutaneous pallor and the CBC shows Hb 7.4 g/dL, Hct 31%, MCV 108. Which of the following determinations will be most useful in deciding the course of action?", "full_answer": "In this question, we are presented with a patient with systemic lupus erythematosus presenting with autoimmune hemolytic anemia. For this, the best option would be to request reticulocytes, however, as it is not among the options, we should request a Coombs' test, so the correct answer is 2. Although they want to confuse us with a MCV 108, so that we think of a megaloblastic anemia, the MCV is not excessively high, the onset of anemia in our patient has been rapid and would not fit with SLE or the form of presentation. Antinuclear antibodies are used for diagnosis, but not for monitoring SLE.", "type": "RHEUMATOLOGY", "options": {"1": "Haptoglobin.", "2": "Coombs test.", "3": "Vitamin B12.", "4": "Antinuclear antibodies.", "5": NaN}, "correct_option": 2, "explanations": {"1": {"exist": false, "char_ranges": [], "word_ranges": [], "text": ""}, "2": {"exist": true, "char_ranges": [[0, 292]], "word_ranges": [[0, 48]], "text": "In this question, we are presented with a patient with systemic lupus erythematosus presenting with autoimmune hemolytic anemia. For this, the best option would be to request reticulocytes, however, as it is not among the options, we should request a Coombs' test, so the correct answer is 2."}, "3": {"exist": true, "char_ranges": [[293, 524]], "word_ranges": [[48, 93]], "text": "Although they want to confuse us with a MCV 108, so that we think of a megaloblastic anemia, the MCV is not excessively high, the onset of anemia in our patient has been rapid and would not fit with SLE or the form of presentation."}, "4": {"exist": true, "char_ranges": [[525, 599]], "word_ranges": [[93, 104]], "text": "Antinuclear antibodies are used for diagnosis, but not for monitoring SLE."}, "5": {"exist": false, "char_ranges": [], "word_ranges": [], "text": ""}}} +{"id": 571, "year": 2022, "question_id_specific": 170, "full_question": "A 75-year-old woman on sertraline treatment who comes to the emergency department with confusional symptoms. There is no evidence of edema and blood pressure is 130/70. The blood test shows Na 126 mEq/l and K 4 mEq/l, natriuria is 45 mEq/l and diuretic intake has been ruled out. Which of the following is the most correct approach?", "full_answer": "We are presented with a patient with moderate euvolemic hyponatremia, suggesting a syndrome of inadequate antidiuresis (SIAD) secondary to sertraline, although it would be necessary to have urinary osmolality to confirm this. Primary adrenal insufficiency produces hypovolemic hyponatremia (incorrect option 1). Diabetes insipidus is the opposite of this pathology, usually presenting with hypernatremia (option 4 incorrect). The question remains whether to treat aggressively with hypertonic saline, or to be more conservative by indicating hydric restriction. With natremia above 125 mmol/l it is not indicated to treat with HSS unless the patient has severe neurological symptoms, which does not seem to be the case (they describe a confusional picture that would be a moderate hyponatremic encephalopathy), and it is not indicated to restore natremia as soon as possible (due to the risk of osmotic demyelination syndrome). Therefore, the correct answer would be 3. What I do not like about this answer is that the second part (\"if natraemia does not rise, switch to slow infusion of saline\") is incorrect, since in this case the use of furosemide, tolvaptan or urea should be considered, not saline (and it does not indicate tonicity either, since isotonic is not the same as hypertonic). In any case, I do not see it as very likely to be overridden.", "type": "NEPHROLOGY", "options": {"1": "It is Addison's disease and corticosteroids should be administered immediately.", "2": "Administer hypertonic saline to restore natremia as soon as possible.", "3": "Indicate water restriction and if natremia does not rise, switch to slow infusion of saline.", "4": "Request brain MRI, since it is probably diabetes insipidus.", "5": NaN}, "correct_option": 3, "explanations": {"1": {"exist": true, "char_ranges": [[226, 311]], "word_ranges": [[32, 41]], "text": "Primary adrenal insufficiency produces hypovolemic hyponatremia (incorrect option 1)."}, "2": {"exist": true, "char_ranges": [[562, 927]], "word_ranges": [[75, 135]], "text": "With natremia above 125 mmol/l it is not indicated to treat with HSS unless the patient has severe neurological symptoms, which does not seem to be the case (they describe a confusional picture that would be a moderate hyponatremic encephalopathy), and it is not indicated to restore natremia as soon as possible (due to the risk of osmotic demyelination syndrome)."}, "3": {"exist": false, "char_ranges": [], "word_ranges": [], "text": ""}, "4": {"exist": true, "char_ranges": [[312, 425]], "word_ranges": [[41, 56]], "text": "Diabetes insipidus is the opposite of this pathology, usually presenting with hypernatremia (option 4 incorrect)."}, "5": {"exist": false, "char_ranges": [], "word_ranges": [], "text": ""}}} +{"id": 466, "year": 2020, "question_id_specific": 91, "full_question": "An 11-year-old, 6-month-old, premenarchal girl with known idiopathic scoliosis. The spine x-ray performed 6 months ago showed a left thoracolumbar curve of 18º (Cobb angle). The current x-ray shows a Cobb angle of 28º, with an iliac crest maturation (Risser's sign) of 2. At this stage of evolution, what is the most appropriate treatment to try to stop the progression of the curve?", "full_answer": "In the juvenile and adolescent forms, treatment depends on the degree of deformity: - 10 - 20°: serial observation. - 20-50°: thermoplastic corsets (Boston, Cheneau or Milwaukee, according to where the apex of the main curve is located). - 40-50°: with Risser less than 3, surgical arthrodesis would be indicated; with Risser greater than or equal to 3, the remaining growth potential of the curve is low and observation may be considered in case it tends to progress in adulthood.", "type": "ORTHOPEDIC SURGERY AND TRAUMATOLOGY", "options": {"1": "Swimming exercises.", "2": "Postural re-education exercises.", "3": "Corrective orthopedic brace.", "4": "Surgical intervention by spinal arthrodesis.", "5": NaN}, "correct_option": 3, "explanations": {"1": {"exist": false, "char_ranges": [], "word_ranges": [], "text": ""}, "2": {"exist": false, "char_ranges": [], "word_ranges": [], "text": ""}, "3": {"exist": true, "char_ranges": [[118, 237]], "word_ranges": [[20, 38]], "text": "20-50°: thermoplastic corsets (Boston, Cheneau or Milwaukee, according to where the apex of the main curve is located)."}, "4": {"exist": true, "char_ranges": [[248, 313]], "word_ranges": [[40, 50]], "text": "with Risser less than 3, surgical arthrodesis would be indicated;"}, "5": {"exist": false, "char_ranges": [], "word_ranges": [], "text": ""}}} +{"id": 175, "year": 2013, "question_id_specific": 53, "full_question": "71-year-old male, retired waiter, non-smoker, who consults for nocturnal snoring. The patient has a body mass index of 31.5 kg/m2 and reports mild postprandial daytime sleepiness (Epworth Sleepiness Scale of 3) with no other relevant symptoms, signs or clinical history. The apnea-hypopnea index obtained in the polysomnography was 18. With these data, what is the most appropriate measure to adopt in this patient?", "full_answer": "With an Apneas-Hypopnea Index of less than 30, and in the absence of striking symptoms of OSAHS and in the absence of risk professions such as professional drivers, etc., the correct thing to do is to control weight and the rest of the hygienic-dietary measures, with periodic follow-up.", "type": "PULMONOLOGY", "options": {"1": "Hygienic-dietary measures and evolution control.", "2": "Uvulopalatopharyngoplasty.", "3": "Nasal CPAP (Continuous Positive Airway Pressure).", "4": "Non-invasive mechanical ventilation.", "5": "Nocturnal oxygen therapy."}, "correct_option": 1, "explanations": {"1": {"exist": true, "char_ranges": [[0, 287]], "word_ranges": [[0, 48]], "text": "With an Apneas-Hypopnea Index of less than 30, and in the absence of striking symptoms of OSAHS and in the absence of risk professions such as professional drivers, etc., the correct thing to do is to control weight and the rest of the hygienic-dietary measures, with periodic follow-up."}, "2": {"exist": false, "char_ranges": [], "word_ranges": [], "text": ""}, "3": {"exist": false, "char_ranges": [], "word_ranges": [], "text": ""}, "4": {"exist": false, "char_ranges": [], "word_ranges": [], "text": ""}, "5": {"exist": false, "char_ranges": [], "word_ranges": [], "text": ""}}} +{"id": 91, "year": 2012, "question_id_specific": 35, "full_question": "A 26-year-old woman presenting with fecal incontinence after a prolonged instrumental delivery. An endoanal ultrasound was performed showing a section of the external anal sphincter of 30º amplitude. The electrophysiological study shows normal innervation. What is the most indicated treatment?", "full_answer": "The answer is surgical sphincteroplasty 3. 1 is not effective, 2 is not effective for an organic lesion with normal electromyography, 4 is not necessary since the lesion is only of the external sphincter, 5 is not indicated in a lesion that can be repaired with surgery with good results.", "type": "GENERAL SURGERY", "options": {"1": "Medical treatment with hygienic and dietary norms.", "2": "Sphincter biofeedback.", "3": "Surgical sphincteroplasty.", "4": "Surgical repair of the pelvic floor.", "5": "Artificial anal sphincter."}, "correct_option": 3, "explanations": {"1": {"exist": true, "char_ranges": [[43, 61]], "word_ranges": [[6, 10]], "text": "1 is not effective,"}, "2": {"exist": true, "char_ranges": [[63, 133]], "word_ranges": [[10, 21]], "text": "2 is not effective for an organic lesion with normal electromyography,"}, "3": {"exist": false, "char_ranges": [], "word_ranges": [], "text": ""}, "4": {"exist": true, "char_ranges": [[134, 204]], "word_ranges": [[21, 34]], "text": "4 is not necessary since the lesion is only of the external sphincter,"}, "5": {"exist": true, "char_ranges": [[205, 288]], "word_ranges": [[34, 50]], "text": "5 is not indicated in a lesion that can be repaired with surgery with good results."}}} +{"id": 65, "year": 2011, "question_id_specific": 218, "full_question": "A 56-year-old man, diabetic and moderately overweight, presents a discrete hepatomegaly. The patient indicates that he is not a regular drinker, only exceptionally does he drink some wine. A liver biopsy shows accumulations of clear cytoplasmic vacuoles in 50% of the hepatocytes. Multiple inflammatory foci with neutrophils, Mallory's hyaline degeneration and fibrosis around the terminal hepatic venules. Point out the correct answer in this case:", "full_answer": "The changes described in the question statement correspond to an early precirrhotic situation. Although alcohol is the most frequent toxicant that causes this condition, it is not the exclusive cause. Diabetes mellitus causes similar hepatic alterations: fat accumulation in hepatocytes, centroacinar fibrosis, inflammatory infiltrate by polymorphonuclear neutrophils and hyaline degeneration of perinuclear mallory. Taking this into consideration, we can rule out the second question (they are intracellular fat accumulations, not glycogen) and the fourth (if diabetes also produces this situation, there is no reason to mistrust the patient). Based on the primum non nocere and not having special relevance for the treatment if it is a focal or generalized disease, it is not necessary to repeat a liver biopsy that would not expand information, discarding the first question. Fortunately for the hypothetical patient, in the initial phase these changes are reversible when the noxa disappears, so we are left with answer number 5.", "type": "PATHOLOGICAL ANATOMY", "options": {"1": "Biopsy should be repeated to see if the changes are diffuse.", "2": "Diagnosis is hepatic glycogenosis associated with diabetes.", "3": "Histological changes are irreversible and will eventually lead to cirrhosis.", "4": "The injury is clearly caused by alcohol despite the patient's refusal.", "5": "It is important to reduce weight and adequately control diabetes to avoid the possible development of cirrhosis."}, "correct_option": 5, "explanations": {"1": {"exist": true, "char_ranges": [[645, 878]], "word_ranges": [[88, 128]], "text": "Based on the primum non nocere and not having special relevance for the treatment if it is a focal or generalized disease, it is not necessary to repeat a liver biopsy that would not expand information, discarding the first question."}, "2": {"exist": true, "char_ranges": [[201, 644]], "word_ranges": [[30, 88]], "text": "Diabetes mellitus causes similar hepatic alterations: fat accumulation in hepatocytes, centroacinar fibrosis, inflammatory infiltrate by polymorphonuclear neutrophils and hyaline degeneration of perinuclear mallory. Taking this into consideration, we can rule out the second question (they are intracellular fat accumulations, not glycogen) and the fourth (if diabetes also produces this situation, there is no reason to mistrust the patient)."}, "3": {"exist": false, "char_ranges": [], "word_ranges": [], "text": ""}, "4": {"exist": true, "char_ranges": [[201, 644]], "word_ranges": [[30, 88]], "text": "Diabetes mellitus causes similar hepatic alterations: fat accumulation in hepatocytes, centroacinar fibrosis, inflammatory infiltrate by polymorphonuclear neutrophils and hyaline degeneration of perinuclear mallory. Taking this into consideration, we can rule out the second question (they are intracellular fat accumulations, not glycogen) and the fourth (if diabetes also produces this situation, there is no reason to mistrust the patient)."}, "5": {"exist": true, "char_ranges": [[921, 1033]], "word_ranges": [[133, 153]], "text": "in the initial phase these changes are reversible when the noxa disappears, so we are left with answer number 5."}}} +{"id": 50, "year": 2011, "question_id_specific": 160, "full_question": "A 5-year-old girl, completely asymptomatic, who comes to the pediatrician for a regular check-up. She consults because her mother was operated on for a thyroid tumor and a grandmother died 10 years ago from a pheochromocytoma. The priority therapeutic attitude we will adopt will be:", "full_answer": "The correct answer is 4. It seems to me to be more of an endocrine question than a pediatric question. Proto-oncogene carriers usually have early thyroid tumors so radical thyroidectomy is usually recommended before the age of 5 years. Obviously I had to look this up...it is not very common in our routine practice.", "type": "PEDIATRICS", "options": {"1": "Exhaustive physical examination with blood pressure control to assess whether the child may have a familial disease.", "2": "Biochemical analysis with calcitonin. If it is normal, no further control is necessary at this age.", "3": "Annual monitoring of calcitonin levels with pentagastrin stimulation and, if elevated, indication of prophylactic thyroidectomy.", "4": "Genetic study of the RET c634 proto-oncogene mutation, and if positive, prophylactic radical thyroidectomy will be performed at this age.", "5": "Genetic study of the RET c634 proto-oncogene mutation, calcitonin levels and FNA (fine needle aspiration) and if positive, radical thyroidectomy will be performed."}, "correct_option": 4, "explanations": {"1": {"exist": false, "char_ranges": [], "word_ranges": [], "text": ""}, "2": {"exist": false, "char_ranges": [], "word_ranges": [], "text": ""}, "3": {"exist": false, "char_ranges": [], "word_ranges": [], "text": ""}, "4": {"exist": true, "char_ranges": [[103, 235]], "word_ranges": [[20, 39]], "text": "Proto-oncogene carriers usually have early thyroid tumors so radical thyroidectomy is usually recommended before the age of 5 years."}, "5": {"exist": false, "char_ranges": [], "word_ranges": [], "text": ""}}} +{"id": 326, "year": 2016, "question_id_specific": 173, "full_question": "A 35-year-old man suffers a high-voltage electrical burn (3000 volts) from direct contact with wire with his left hand. On admission he presents flexion contracture of the hand, pallor of the fingers and absence of radial and ulnar pulse on palpation What is the emergency invasive measure to be performed?", "full_answer": "3. In the case of suspected compartment syndrome, the urgent invasive measure to perform would be to measure the intracompartmental pressure and, depending on the result, proceed with treatment.", "type": "CRITICAL, PALLIATIVE AND EMERGENCY CARE", "options": {"1": "Axillary block with catheter.", "2": "Decompressive scarotomy.", "3": "Intracompartmental pressure monitoring.", "4": "Scarectomy.", "5": NaN}, "correct_option": 3, "explanations": {"1": {"exist": false, "char_ranges": [], "word_ranges": [], "text": ""}, "2": {"exist": false, "char_ranges": [], "word_ranges": [], "text": ""}, "3": {"exist": true, "char_ranges": [[3, 194]], "word_ranges": [[1, 29]], "text": "In the case of suspected compartment syndrome, the urgent invasive measure to perform would be to measure the intracompartmental pressure and, depending on the result, proceed with treatment."}, "4": {"exist": false, "char_ranges": [], "word_ranges": [], "text": ""}, "5": {"exist": false, "char_ranges": [], "word_ranges": [], "text": ""}}} +{"id": 354, "year": 2016, "question_id_specific": 166, "full_question": "A pregnant woman, 10 weeks pregnant, comes to the emergency room with a blood pressure of 160/105 mmHg. She was in good general condition, with only mild headache, which was the reason for taking her blood pressure. After 4 hours of rest she presented with a blood pressure of 150/95mmHg. A complete blood count is normal and proteinuria is negative. What type of hypertension is present?", "full_answer": "Chronic hypertension is defined as hypertension that is present before pregnancy or that is diagnosed before the 20th week of gestation. In this case, the patient is 10 weeks pregnant, so the correct answer is 3.", "type": "GYNECOLOGY AND OBSTETRICS", "options": {"1": "Moderate preeclampsia.", "2": "Pregnancy-induced hypertension.", "3": "Chronic hypertension.", "4": "Eclampsia.", "5": NaN}, "correct_option": 3, "explanations": {"1": {"exist": false, "char_ranges": [], "word_ranges": [], "text": ""}, "2": {"exist": false, "char_ranges": [], "word_ranges": [], "text": ""}, "3": {"exist": true, "char_ranges": [[0, 212]], "word_ranges": [[0, 36]], "text": "Chronic hypertension is defined as hypertension that is present before pregnancy or that is diagnosed before the 20th week of gestation. In this case, the patient is 10 weeks pregnant, so the correct answer is 3."}, "4": {"exist": false, "char_ranges": [], "word_ranges": [], "text": ""}, "5": {"exist": false, "char_ranges": [], "word_ranges": [], "text": ""}}} +{"id": 531, "year": 2021, "question_id_specific": 58, "full_question": "Faced with a patient presenting with an ulcerovegetating lesion on the mandibular gingiva at the level of the molar region of 5 months of evolution, the appearance of hypoesthesia in the mentonian region requires ruling out:", "full_answer": "Mentonian paresthesias (Roger's sign) are associated with bone metastases (it is not paraneoplastic but direct infiltration). In this case what you describe is an oral tumor growing locally and infiltrating the mandibular canal, I believe.", "type": "ONCOLOGY", "options": {"1": "Paraneoplastic neuropathy.", "2": "Diagnosis of sarcoidosis with \"on the scale\" facial paralysis.", "3": "Herpes virus infection in immunosuppressed patient.", "4": "Diagnosis of carcinoma with infiltration of the mandibular canal.", "5": NaN}, "correct_option": 4, "explanations": {"1": {"exist": false, "char_ranges": [], "word_ranges": [], "text": ""}, "2": {"exist": false, "char_ranges": [], "word_ranges": [], "text": ""}, "3": {"exist": false, "char_ranges": [], "word_ranges": [], "text": ""}, "4": {"exist": true, "char_ranges": [[0, 239]], "word_ranges": [[0, 35]], "text": "Mentonian paresthesias (Roger's sign) are associated with bone metastases (it is not paraneoplastic but direct infiltration). In this case what you describe is an oral tumor growing locally and infiltrating the mandibular canal, I believe."}, "5": {"exist": false, "char_ranges": [], "word_ranges": [], "text": ""}}} +{"id": 93, "year": 2012, "question_id_specific": 31, "full_question": "A 17-year-old boy with Crohn's disease with colonic involvement of 2 years of evolution, in maintenance treatment with azathioprine, consults for the appearance for 5 days of subcutaneous purplish red, hot, painful, bilateral, pretibial localization nodules, associated with an increase in the number of stools and abdominal pain. The most appropriate approach in this case is:", "full_answer": "2 and 4 are discarded alone. They are explaining erythema nodosum, so we would recommend relative rest (but antidepressants do not make any sense, so we also rule out (1)). Although erythema nodosum can be secondary to a malignant process, it is more frequent that it is due to Crohn's disease itself (moreover, in a 17-year-old boy it would be extremely rare), so if the picture is also accompanied by a flare-up of inflammatory bowel disease, the last option (5) seems the most correct.", "type": "DERMATOLOGY, VENEREOLOGY AND PLASTIC SURGERY", "options": {"1": "Recommend relative rest and warm cloths on both legs and add antidepressant treatment.", "2": "Biopsy skin areas away from the injured areas and prescribe entry opioid analgesics.", "3": "Suspect the existence of a malignant intestinal tumor lesion as a trigger of the cutaneous process.", "4": "Suspect bilateral lower extremity ischemia of drug origin.", "5": "Adjustment of bowel disease treatment."}, "correct_option": 5, "explanations": {"1": {"exist": true, "char_ranges": [[29, 172]], "word_ranges": [[6, 30]], "text": "They are explaining erythema nodosum, so we would recommend relative rest (but antidepressants do not make any sense, so we also rule out (1))."}, "2": {"exist": false, "char_ranges": [], "word_ranges": [], "text": ""}, "3": {"exist": true, "char_ranges": [[173, 360]], "word_ranges": [[30, 62]], "text": "Although erythema nodosum can be secondary to a malignant process, it is more frequent that it is due to Crohn's disease itself (moreover, in a 17-year-old boy it would be extremely rare),"}, "4": {"exist": false, "char_ranges": [], "word_ranges": [], "text": ""}, "5": {"exist": true, "char_ranges": [[365, 488]], "word_ranges": [[63, 84]], "text": "if the picture is also accompanied by a flare-up of inflammatory bowel disease, the last option (5) seems the most correct."}}} +{"id": 390, "year": 2016, "question_id_specific": 188, "full_question": "An 80-year-old man is brought to the ER by the police to find him at night, on the street, several blocks from his home, somewhat lost. He tells the ER doctor \"I feel fine, I was walking and I got lost\". The examination, by systems, is anodyne. When asked about his medication, he does not remember. His history includes glipizide, atenolol and acenocoumarol. She has been to the ER 3 times in the last three months for \"dizziness\", low back pain and an INR of 5. She reports independence in basic and instrumental activities of daily living. There is no evidence of family or close friends. Several references from the social work in the area indicate that she refuses medical or nursing aids or visits. In the physical examination, only temperature of 37.7, BMI of 16, BP of 160/90, HR of 90 and SO2 of 99% stand out. He has global loss of muscle mass. He is dressed in his robe and smells of urine. He refuses to answer Minimental's questions. Laboratory tests include 13,000 leukocytes, blood glucose 300, glycosylated hemoglobin 11%, INR 0.9, urine over 50 leukocytes/field, nitrites+ and urine culture > 100,000 Gram negative bacilli. He is admitted to the hospital, receives antibiotics, insulin and restarts acenocoumarol with good evolution. He is stable and wants to go home. The ward nurse says he is unable to self-administer insulin. Which of the following is the next step in determining the patient's safety at home?", "full_answer": "Assessing capacity is not about taking away the patient's autonomy, but making sure that the patient understands and is aware of the consequences of his decisions. In fact, it is a protective measure for the most fragile patients to ensure the best care. This is not done by a psychiatrist, but by the attending physician. There are several tests or semi-structured interviews to assess the capacity of patients, and all physicians should be familiar with these instruments. It is important that it is the attending physician who performs this assessment, because he/she is the one who previously informs the patient (informed consent) of the therapeutic plan and is the one who is best able to assess whether the patient has understood everything: what the medical problem is, what treatment is proposed, the alternatives if any, whether to accept or reject any of them and why, the consequences of making these decisions (also those of not wanting treatment), even assessing psychopathology susceptible to an ICP to Psychiatry.", "type": "PSYCHIATRY", "options": {"1": "Refer to Primary Care and social work in the area to determine safety at home.", "2": "Formal evaluation of the capacity to make decisions concerning their health.", "3": "Evaluation to rule out dementia.", "4": "Evaluation to rule out depression.", "5": NaN}, "correct_option": 2, "explanations": {"1": {"exist": false, "char_ranges": [], "word_ranges": [], "text": ""}, "2": {"exist": true, "char_ranges": [[173, 254]], "word_ranges": [[28, 43]], "text": "it is a protective measure for the most fragile patients to ensure the best care."}, "3": {"exist": false, "char_ranges": [], "word_ranges": [], "text": ""}, "4": {"exist": false, "char_ranges": [], "word_ranges": [], "text": ""}, "5": {"exist": false, "char_ranges": [], "word_ranges": [], "text": ""}}} +{"id": 577, "year": 2022, "question_id_specific": 105, "full_question": "A 45-year-old male brought to the consultation by his relatives for a one-year history of memory loss, which has progressed to the point of needing help with some activities of daily living. He has a history of migraines and some episode of self-limited neurological focality for which he has never consulted. The neuropsychological examination is compatible with an initial dementia and the brain MRI shows a severe leukoencephalopathy. Which test should be ordered?", "full_answer": "They describe a characteristic case of CADASIL (Cerebral Autosomal Dominant Arteriopathy with Sub-cortical Infarcts and Leukoencephalopathy). It is a genetic disease of autosomal dominant inheritance by mutation of the NOTCH3 gene. This mutation causes vascular involvement of small vessel that generates severe leukoencephalopathy early, which generates various symptoms. Migraines, cerebrovascular disease and cognitive impairment at early ages stand out, as shown in the clinical case of the question.", "type": "NEUROLOGY", "options": {"1": "Lumbar puncture to analyze amyloid and tau protein in CSF.", "2": "Genetic study for presenilin 1 (PSEN1).", "3": "Genetic study for NOTCH3.", "4": "Genetic study for progranulin.", "5": NaN}, "correct_option": 3, "explanations": {"1": {"exist": false, "char_ranges": [], "word_ranges": [], "text": ""}, "2": {"exist": false, "char_ranges": [], "word_ranges": [], "text": ""}, "3": {"exist": true, "char_ranges": [[0, 231]], "word_ranges": [[0, 31]], "text": "They describe a characteristic case of CADASIL (Cerebral Autosomal Dominant Arteriopathy with Sub-cortical Infarcts and Leukoencephalopathy). It is a genetic disease of autosomal dominant inheritance by mutation of the NOTCH3 gene."}, "4": {"exist": false, "char_ranges": [], "word_ranges": [], "text": ""}, "5": {"exist": false, "char_ranges": [], "word_ranges": [], "text": ""}}} +{"id": 615, "year": 2022, "question_id_specific": 118, "full_question": "A 41-year-old male consults for gonalgia of several days' evolution. During the examination, the Thessaly test is performed (pain with internal and external rotation movements with the knee flexed), which is positive. Which of the following lesions is more probable?", "full_answer": "Answer 1 correct: The Thessaly test is a meniscal exploration maneuver. The statement focuses exclusively on this part of the examination and also explains that it is indeed positive, which leads to the suspicion of a meniscal lesion. \"The Thessaly test is a dynamic reproduction of load transmission in the knee joint and is performed at 5° and 20° of flexion. The examiner supports the patient by holding his or her outstretched hands while the patient stands flatfooted on the floor. The patient then rotates his or her knee and body, internally and externally, three times, keeping the knee in slight flexion (5°). Then the same procedure is carried out with the knee flexed at 20°. Patients with suspected meniscal tears experience medial or lateral joint-line discomfort and may have a sense of locking or catching. The theory behind the test is that, with this maneuver, the knee with a meniscal tear is subjected to excessive loading conditions and almost certainly will have the same symptoms that the patient reported. The test is always performed first on the normal knee so that the patient may be trained, especially with regard to how to keep the knee in 5° and then in 20° of flexion and how to recognize, by comparison, a possible positive result in the symptomatic knee.\" Answer 2 incorrect: At no time do they describe any exploration maneuvers of the anterior cruciate ligament, nor do they mention any trauma that may have injured it. Answer 3 incorrect: At no time do they describe any exploration maneuvers of the posterior cruciate ligament, nor do they mention any trauma that could have injured it. Wrong answer 4: Although it could occur, it is not to be expected in a 41 year old person with an acutely established clinical condition.", "type": "TRAUMATOLOGY", "options": {"1": "Meniscal injury.", "2": "Injury due to rupture of the anterior cruciate ligament.", "3": "Injury due to rupture of the posterior cruciate ligament.", "4": "Injury due to degenerative arthropathy.", "5": NaN}, "correct_option": 1, "explanations": {"1": {"exist": true, "char_ranges": [[0, 234]], "word_ranges": [[0, 38]], "text": "Answer 1 correct: The Thessaly test is a meniscal exploration maneuver. The statement focuses exclusively on this part of the examination and also explains that it is indeed positive, which leads to the suspicion of a meniscal lesion."}, "2": {"exist": true, "char_ranges": [[1289, 1454]], "word_ranges": [[218, 246]], "text": "Answer 2 incorrect: At no time do they describe any exploration maneuvers of the anterior cruciate ligament, nor do they mention any trauma that may have injured it."}, "3": {"exist": true, "char_ranges": [[1455, 1623]], "word_ranges": [[246, 274]], "text": "Answer 3 incorrect: At no time do they describe any exploration maneuvers of the posterior cruciate ligament, nor do they mention any trauma that could have injured it."}, "4": {"exist": true, "char_ranges": [[1624, 1761]], "word_ranges": [[274, 299]], "text": "Wrong answer 4: Although it could occur, it is not to be expected in a 41 year old person with an acutely established clinical condition."}, "5": {"exist": false, "char_ranges": [], "word_ranges": [], "text": ""}}} +{"id": 456, "year": 2018, "question_id_specific": 200, "full_question": "A 67-year-old man, diabetic and asthmatic, with primary open-angle glaucoma in both eyes. He has an ocular pressure of 34 mmHg in his right eye and 31 mmHg in his left eye. Which of the following pharmacological groups is the most indicated as first choice as hypotensive treatment?", "full_answer": "We are asked about the first choice treatment for glaucoma. In the past, beta-blockers were the first choice, but nowadays they have been displaced by the group that lowers intraocular pressure the most: prostaglandins. Their efficacy and more convenient dosage (once a day) generally make them the group of choice. Beta-blockers would remain as a second choice in cases in which the side effects of prostaglandins (change in eye color, increase in eyelash size) make us initially leave them aside. Since the patient is asthmatic, we should rule out beta-blockers (wrong answer 2). Also, since we need a significant drop in intraocular pressure because the numbers are high (34 and 31), we can also rule out carbonic anhydrase inhibitors and alpha-2 agonists (options 1 and 3 incorrect). And opt for the group that can lower intraocular pressure the most. Answer 4 correct. And then a couple of nuances of the statement. It would be more correct to talk about intraocular pressure and not \"ocular pressure\". And the wording could be improved so as not to repeat the word \"as\" in the last sentence.", "type": "OPHTHALMOLOGY", "options": {"1": "Carbonic anhydrase inhibitors.", "2": "Beta-blockers.", "3": "Alpha-2 agonists.", "4": "Prostaglandin derivatives.", "5": NaN}, "correct_option": 4, "explanations": {"1": {"exist": true, "char_ranges": [[582, 787]], "word_ranges": [[93, 127]], "text": "Also, since we need a significant drop in intraocular pressure because the numbers are high (34 and 31), we can also rule out carbonic anhydrase inhibitors and alpha-2 agonists (options 1 and 3 incorrect)."}, "2": {"exist": true, "char_ranges": [[499, 581]], "word_ranges": [[80, 93]], "text": "Since the patient is asthmatic, we should rule out beta-blockers (wrong answer 2)."}, "3": {"exist": true, "char_ranges": [[582, 787]], "word_ranges": [[93, 127]], "text": "Also, since we need a significant drop in intraocular pressure because the numbers are high (34 and 31), we can also rule out carbonic anhydrase inhibitors and alpha-2 agonists (options 1 and 3 incorrect)."}, "4": {"exist": true, "char_ranges": [[60, 219]], "word_ranges": [[10, 34]], "text": "In the past, beta-blockers were the first choice, but nowadays they have been displaced by the group that lowers intraocular pressure the most: prostaglandins."}, "5": {"exist": false, "char_ranges": [], "word_ranges": [], "text": ""}}} +{"id": 484, "year": 2020, "question_id_specific": 175, "full_question": "An 18-year-old man arrives at the hospital emergency department transported by a basic life support unit. He has been hit by a car while riding a bicycle. On arrival he has a heart rate of 115 bpm, a blood pressure of 110/75 mmHg, a palpable radial pulse, a capillary refill of 2.5 seconds and a respiratory rate of 25 rpm. What do these data tell us:", "full_answer": "Technically the patient is in a grade II hemorrhage according to the ATLS classification, in principle this means a loss of between 750 and 1500mL of blood and the indicated action is resuscitation with crystalloids; however, in the guidelines for the care of severe polytraumatized patients, the early use of blood products is increasingly recommended, especially in borderline cases like this one, and in young patients, who may not be able to translate the severity of the blood loss until it becomes much more significant. I would therefore hesitate between options 1 and 2.", "type": "CRITICAL CARE", "options": {"1": "The patient has lost 15 to 30% of blood volume and requires volume replacement.", "2": "She has lost more than 30% of volemia and needs volume and blood replacement.", "3": "He has lost more than 40% of blood volume and will require surgical treatment.", "4": "Has lost more than 50% of volemia and requires volume and blood replacement.", "5": NaN}, "correct_option": 1, "explanations": {"1": {"exist": false, "char_ranges": [], "word_ranges": [], "text": ""}, "2": {"exist": false, "char_ranges": [], "word_ranges": [], "text": ""}, "3": {"exist": false, "char_ranges": [], "word_ranges": [], "text": ""}, "4": {"exist": false, "char_ranges": [], "word_ranges": [], "text": ""}, "5": {"exist": false, "char_ranges": [], "word_ranges": [], "text": ""}}}